Sei sulla pagina 1di 83

Critical reasoning practice

This file contains over 100 critical reasoning questions with


full answers and explanations.
Good luck on your test.

Between 1990 and 1998, occupancy in Branson Hospital averaged 79 percent of


capacity, while admission rates remained constant, at an average of 9 admissions
per 100 beds per year. Between 1998 and 2002, however, occupancy rates
increased to an average of 85 percent of capacity, while admission rates declined
to 8 per 100 beds per year.
Which of the following conclusions can be most properly drawn from the info
given above?
A. The average stay for Branson Hospital residents rose between 1998 and 2002.
B. The proportion of children staying in Branson hospital was greater in 2002
than in 1990.
C. Hospital admission rates tend to decline whenever occupancy rates rise.
D. Hospitals built prior to 1998 generally had fewer beds than did hospitals built
between 1998 and 2002.
E. The more beds a nursing home has, the higher its occupancy rate is likely to
be.

The best answer is A. Choice A receives support from the fact stated above: between
1998 and 2002, Branson hospital occupancy rates rose though admission rats
declined.

American customers expect high quality. When the quality of a manufactured


product is raised, it in turn raises customer expectations. A company that
believes that the quality of its products is satisfactory will soon discover that its
customers are not similarly satisfied. The goal of Sunnybrook Corporation is to
meet or exceed customer expectations.
Which of the following must be true on the basis of the statements above?
A. Sunnybrooks competitors will succeed in attracting customers only if those
competitors adopt Sunnybrooks goal as their own.
B. A company that does not correctly forecast the expectations of its customers
is certain to fail in advancing the quality of its products.
C. It is possible to meet the goal of Sunnybrook Corporation only if the quality of
their products can be continually improved upon.
D. If a company becomes satisfied with the quality of the products it
manufactures, then the quality is likely to decline.

E. Sunnybrooks customers are currently satisfied with the quality of its products.
The best answer is C. Sunnybrook wishes to meet customer expectations,
however these expectations are always being raised. Whatever the quality of
Sunnybrooks products, it will be possible to meet their goal only by continually
improving their products.

According to a review of 38 studies of patients suffering from water retention, a large


majority of the patients reported that fasting eased their suffering considerably. Yet
fasting is not used to treat water retention even though the conventional medications
often have serious side effects.
Which of the following, if true, best explains the fact that fasting is not used as a
treatment for water retention?
A. For a small percentage of patients with water retention, fasting induces a
temporary sense of nausea
B. Getting patients with water retention to fast regularly is more difficult than getting
healthy patients to do so.
C. Fasting regularly over a long period of time can lead to temporary impairment of
balance comparable to that induced by consuming several ounces of alcohol.
D. The dramatic shifts in water retention connected with fasting have not been traced
to particular changes in brain chemistry.
E. The water retention returns in full force as soon as the fast is broken by even a
small meal.
The best answer is E. The longer one fasts, the more urgently a patient will need to
eat. According to choice E, the water retention would then return in full force. This
would explain why fasting is not used to treat water retention.

The more frequently employees that take a break for guided relaxation exercises
during working hours each week, the fewer sick days they take. Even workers who
take a guided relaxation break once a week during working hours take less sick leave
than those who do not perform relaxation. Therefore, if companies started guided
relaxation programs, the absentee rate in those companies would decrease
significantly.
Which of the following, if true, most seriously weakens the argument above?
A. Employees who do guided relaxation exercises during working hours occasionally
fall asleep for short periods of time after the exercises.
B. Employees who are frequently absent are the least likely to cooperate with or to
join a corporate relaxation program.
C. Employees who do guided relaxation exercises only once a week in their
companys fitness program usually also do relaxation exercises at home.
D. Employees who do guided relaxation exercises in their companys relaxation
program use their working time no more productively than those who do not do
relaxation exercises.
E. Employees do guided relaxation exercises during working hours take slightly
longer lunch breaks than employees who do not do relaxation exercises.
The best answer is B. Even supposing that increasing the frequency of relaxation
leads to less sick time being taken, starting a company-supported relaxation program
might not produce significantly lowered absentee rates if employees who are
frequently absent would not cooperate with such a program. Choice B says that such
cooperation is unlikely and is the best answer.

Home insurance premiums are typically set after an appraiser assigns an official
assessed value. Reassessments should be frequent in order to remove distortions that
arise when property value changes at differential rates. In actual fact, however,
properties are reassessed only when they stand to benefit the insurance company. In
other words, a reassessment takes place when the most likely outcome is in increase
in insurance premiums to the homeowner.
Which of the following, if true, describes a situation in which a reassessment should
occur but in unlikely to do so?
A. Property values have risen sharply and uniformly.
B. Property values have risen everywhere some very sharply, some moderately.
C. Property values have on the whole risen sharply; yet some have dropped slightly.
D. Property values have on the whole dropped significantly; yet some have risen
slightly.
E. Property values have dropped significantly and uniformly.
The best answer is D. Distortions occur when property values change at differential
rates. If most property values have dropped significantly, but some have risen slightly,
a reassessment should occur but is unlikely to do so since it will not benefit the
insurance companies.

Although perioral dermatitis rashes are believed to be caused by reactions to Sodium


Laurel Sulfate (SLS) found in shampoos and other personal care products, instructing
patients to eliminate all products with SLS frequently does not stop the perioral
dermatitis. Obviously, some other cause of perioral dermatitis besides reactions to
SLS must exist.
Which of the following, if true, would most seriously weakens the conclusion above?
A. Many personal care products elicit an allergic skin response only after several
days, making it very difficult to observe links between specific products patients use
and the perioral dermatitis they develop.
B. Skin allergies affect many people who never develop the symptom of perioral
dermatitis.
C. Many patients report that the personal care products that cause them perioral
dermatitis are among the products they most enjoy using.
D. Very few patients have allergic skin reactions as children and then live rash-free
adult lives once they have eliminated products to which they have been demonstrated
to be allergic.
E. Very rarely do personal care products cause patients to suffer a symptom more
severe than that of perioral dermatitis rashes.
The best answer is A. If it is difficult to determine which personal care products cause
perioral dermatitis rashes, then some products that cause allergic reactions might not
have been demonstrated to do so. Hence, if choice A is true, eliminating products that
have been demonstrated to cause rashes might not eliminate the rashes, even if
allergies to these types of products are the only cause of perioral dermatitis.

In prenatal testing for spina bifida, a life threatening disease, a false positive result
indicates that an fetus has spina bifida when, in fact, it does not; a false negative result
indicates that a fetus does not have spina bifida when, in fact, it does. To detect spina
bifida most accurately, physicians should use the laboratory test that has the lowest
proportion of false positive results.
Which of the following, if true, gives the most support to the recommendation above?
A. All laboratory tests to detect spina bifida have the same proportion of false
negative results.
B. The laboratory test that has the lowest proportions of false positive results causes
the same minor side effects as do the other laboratory tests used to detect spina bifida.
C. In treating spina bifida patients, it is essential to begin treatment as early as
possible, since even a week of delay can result in loss of life.
D. The proportion of inconclusive test results is equal for all laboratory tests used to
detect spina bifida.
E. The accepted treatment for spina bifida does not have damaging side effects.
The best answer is A. Only if all tests have the same proportion of false negative
results does the one with the lowest proportion of false positives become the most
accurate.

Health club membership has increased dramatically over the last five years. In order
to take advantage of this increase, Fitness Express plans to open more of the same
types of classes available during the week, while continuing its already very extensive
advertising in newspapers and on the radio.
Which of the following, if true, provides most support for the view that Fitness
Express cannot increase membership to its gyms by adopting the plan outlined above?
A. Although it fills all of the classes it opens, Fitness Expresss share of all health
club memberships has declined over the last five years.
B. Fitness Express number of classes offered to their clients has declined slightly
over the last year.
C. Advertising has made the name of Fitness Express widely known, but few
customers know that Fitness Express health clubs also off acupuncture and shiatsu
treatments.
D. Fitness Expresss health clubs is one of three clubs that have together accounted
for 88% of health club memberships.
E. Despite a slight decline in membership price, sales of Fitness Express
memberships have fallen in the last five years.
The best answer is E. According to the information is choice E, Fitness Express has
already failed to participate in the industry trend of greater sales despite their
advertising campaign. Opening more of the same types of classes would therefore be
unlikely to create more membership sales for Fitness Express.

In a psychological experiment conducted at Southbay University, groups of men with


various levels of education read stories in which people caused harm, some of them
doing so intentionally, and some accidentally. When asked about appropriate
penalization for those who had caused harm, the less educated men, unlike the
educated ones, assigned punishments that did not vary according to whether the harm
was done intentionally or accidentally. Uneducated men, then, do not regard peoples
intentions as relevant to penalization.
Which of the following, if true, would most seriously weaken the conclusion above?
A. In these stories, the severity of the harm produced was clearly stated.
B. In interpreting these stories, the listeners had to draw on a relatively feminine sense
of human psychology in order to tell whether harm was produced intentionally or
accidentally.
C. Relatively uneducated men are as likely to produce harm unintentionally as are
more educated men.
D. The more educated men assigned penalization in a way that closely resembled the
way women had assign penalization in a similar experiment.
E. The less educated men assigned penalization that varied according to the severity
of the harm done by the agents in the stories.
The best answer is B. Choice B indicates that less educated men might be unable to
tell whether the harm in the stories was produced intentionally. Thus, even if less
educated men do regard peoples intentions as relevant, they might be unable to apply
this criterion here. B, therefore, undermines the conclusions support.

More sports journals are sold in Hornby than in Milston. Therefore, the residents of
Hornby are better informed about major sporting events than are the residents of
Milston.
Each of the following, if true, weakens the conclusion above EXCEPT:
A. Hornby has a larger population than Milston.
B. Most residents of Milston work in Hornby and buy their reading material there.
C. The average newsstand price of journals sold in Hornby is lower than the average
price of journals sold in Milston.
D. A monthly journal restricted to the coverage of local events is published in
Hornby.
E. The average resident of Hornby spends less time reading sports journals than does
the average resident of Milston.
The best answer is C. The price differential noted in C might help to explain the
difference in sales, but it does not undermine the conclusion based on the difference.
Therefore, C is the best answer.

Which of the following, best completes the passage below?


At last months symposium on the increasing air pollution affecting the Beaumont
forest, most participating members favored uniform controls on the quality of exhaust
fumes, whether of not specific environmental damage could be attributed to a
particular source of fumes. What must, of course, be shown, in order to avoid
excessively restrictive controls, is that __________ .
A. any uniform controls that are adopted are likely to be implemented without delay
B. environmental damage already inflicted on the Beaumont forest is reversible
C. the members favoring uniform controls are those representing bodies that generate
the largest quantities of exhaust fumes
D. all of any given pollutant that is to be controlled actually reaches Beaumont forest
at present
E. any substance to be made subject to controls can actually cause environmental
damage
The best answer is E. If a subject that causes no environmental damage were subject
to controls, those controls would be more restrictive than necessary.
Although the new cochleae manufactured by Medotech will cost more than twice as
much as the cochlear implants now in use, Medotech implants should still be costeffective. Not only will surgery and recovery time be reduced, but Medotech cochlear
replacements should last longer, thereby reducing the need for further hospital stays.
Which of the following must be studied in order to evaluate the argument presented
above?

A. The amount of time a patient spends in surgery versus the amount of time spent
recovering from surgery.
B. The amount by which the cost of producing Medotech cochlear replacements has
declined with the introduction of the new technique for producing them.
C. The degree to which the use of Medotech cochlear replacements is likely to reduce
the need for repeat surgery when compared with the use of the replacements now in
use.
D. The degree to which the use of Medotech replacements are more carefully
manufactured than are the replacements now in use,
E. The amount by which Medotech will drop in cost as the production procedures
become standardized and applicable on a larger scale.
The best answer is C. To evaluate the argument it must be determined whether these
savings will compensate for the increased cost. Therefore, study of the expected
reduction in the need for further hospital stays is needed.

Clarinet reeds often lose their freshness become less responsive after a few weeks
of intense use. One clarinetist has hypothesized that a buildup of oil, rather than
changes in the material properties of the reed, were responsible.
Which of the following investigations is most likely to yield significant information
that would help to evaluate the hypothesis?
A. Determining what kind of wood is used to make the reeds used by jazz clarinetists.
B. Determining whether jazz clarinetists make their reeds lose their freshness faster
than do classical clarinetists.
C. Determining whether identical lengths of reeds, of the same thickness, lose their
freshness at different rates when put onto various brands of clarinets.
D. Determining whether rubbing various substances on the new reed causes them to
lose their freshness.
E. Determining whether a fresh reed and one that has lost its freshness produce
different qualities of sound
The best answer is D. The hypothesis has two parts: first, that intense use does not
bring material changes that cause the reed to lose its freshness and, second, that oil
causes the phenomenon. The experiment suggested in choice D directly test this

hypothesis by contaminating strings that are known to have their original material
properties.

Wheat has more nutritional value than rye. But since oats have more nutritional value
than rice, it follows that wheat has more nutritional value than rice.
Any of the following, if introduced into the argument as an additional premise, makes
the argument above logically correct EXCEPT:
A. Wheat and oats have the same nutritional value.
B. Rye has more nutritional value than oats.
C. Rye has more nutritional value than rice.
D. Rye and oats have the same nutritional value.
E. Oats have more nutritional value than wheat
The best answer is E. The question asks for a different premise that does not make
the argument logically correct. Choice A leaves open the possibility that the
conclusion of the argument is false.
A famous painter has recently won a lawsuit against a major food manufacturer for
commissioning a graphic designer to design the packaging of its chocolate chip
cookies in the painters disctincive style. As a result of the lawsuit, manufacturers
will stop asking graphic artists to copy distinctive painting styles. Therefore, the cost
of package design will rise because employing the services of known artists cost more
than those of graphic designer that imitate their style.
The conclusion above is based on which of the following assumptions?
A. Most people are unable to distinguish a famous artists illustration from that of a
good graphic artist imitating the painters style.
B. Manufacturers will use well-known painting styles in their packaging.
C. The original version of some well-knows paintings are unavailable for use in
packaging.
D. Manufacturuers will continue to use graphic artists to imitate the artistic style of
famous painters.
E. Packaging using a famous painters style usually sell better than packaging using
the artistry of imitators.

The best answer is B. If choice E were not assumed, the costs of the services of the
famous painters would not be siad to affect packaging costs. Since packaging costs
are, however, projected to rise because of the relatively high cost of famous painters
services, choice E is assumed and is the best answer.

Excavations on the now uninhabited isle of Kelton reveal a scene typical of towns
decimated by volcanic ash. Archaeologists have hypothesized that the destruction
was due to volcanic activity known to have occurred in the vicinity of the island in
160 B. C.
Which of the following, if true, most strongly supports the archarologist hypothesis?
A. No coins minted after 160 B.C. were found in Kelton , but coincs minted before
that year were found in abundance.
B. Pieces of gold and pearl jewelery that are often found in graves dating from years
preceding and following 160 B.C. were also found in several graves on the island.
C. Most modern histories of the region mention that several major volcanoe
erruptions occurred near the island in 160 B.C.
D. Several small jugs carved in styles poplular in the region in the certury between
200 B.C. and 100 B.C. were found in Kelton.
E. Drawings of the styles that was definitely used in the region after 160 B.C. were
found in Kelton.
The best answer is A. The archeologist hypothesized that Kelton was decimated by
volcanic ash in 160 B.C. Since A provides evidence that 160 B.C. was the date when
life in Kelton was disrupted, A supports the hypothesis.

Many community colleges suffer declining enrollments during periods of economic


ressession. At government retraining programs, which are not provided free of
charge, enrollment figures boom during these periods when many people have less
money and there is more competition for jobs.
Each of the following, if true, helps to explain the enrollment increases in government
retraining programs above EXCEPT:
A. During periods of economic slowdown, government retraining programs are more
likely that community colleges to prpare their students for the jobs that are still
available.
B. During periods of economic prosperity, grauates of government retraining
programs often continue their studies in community colleges.
C. Tuition at most government retraining programs is a fraction of that at community
colleges.
D. Government retraining programs devote more resources than do other colleges to
attracting those studnets especially affected by economic slowdowns.
E. Students at government retraining programs, but not those at most community
colleges, can control the cost of their studies by choosing the number of courses they
take each term.
The best answer is B. Choice B might explain the decreased enrollment at
community colleges during the slowdown, but because it deals with graduates of

government retraining programs it cannot explain why enrollment at these colleges


might increase.

Parents of some of the children in a particular class have claimed that the teacher is
not objective and favors his male students with higher grades. But the record shown
that 92% of the female students received a passing grade in this teachers class. This
record demonstrates that the teacher has not discriminated against women when
assigning grades.
The argument above is flawed in that it ignored the possibility that
A. a large number of the teachers student were in his class the previous year.
B. many teachers find it difffucult to be objective when assigning grades to male and
female students.
C. the evidence shows that more than 92% of the female students should have
received a passing grade.
D. the majority of tests written by female students and that have been rechecked by
another teacher were given a higher grade by the second teacher.
E. the teacher is biased against female students in the case of only some of the
subjects he teachers.
The best answer is C. The flaw in the argument is that it assumes erroneously that a
large percentage of female students who pass this teachers class demonstrates
absence of discriminatory behaviour against women on the part of the teacher who
gave the grade. Choice E exposes this flaw by pointing out that the judge may well
have failed to pass a female student in cases where evidence shows that the student
should have passed.

The number of young adults that are illiterate has dropped significantly in a certain
county over the last fifteen years. Education officials attribute this decrease entirely
to improved conditions in the schools, which made for a better teaching environment,
reducing the level of illiteracy.
Which of the following, if true, would most seriously weaken the education officials
explanation for the lower incidence of the disease?
A. Many similar improvements in school conditions have been made over the last
twenty-five years in the county.
B. Home-schooling has not been more prevalent among the illiterate as compared to
people who are not illiterate.
C. Because of a new assessment scale, many people who until this year would have
been considered illiterate are now considered low-level readers.
D. The same percentage of the population has been tested every year for the last 30
years.
E. The conditions in the schools were brought up to the standards of the neighboring
counties twenty years ago.
The best answer is C. The education officials assume that the decrease in the number
of people who are illiterate reflects a diminution in cases of illiteracy. By pointing out
that this assumption is false, choice C undermines the officials explanation.

In many hospitals, more and more monitoring is being carried out by automated
equipment instead of human employees who previous carried out the work. This is
done in an attempt to save hospital expenses. However, many employees who lose
their jobs to automation will need government assistance to get by, and the same
corporations that are laying people off will eventually pay for that assistance through
increased taxes and unemployment insurance payments.
Which of the following, if true, most strengthens the authors argument?
A. Most of the new jobs created by automated equipment pay less than the jobs that
were eliminated by automated equipment did.
B. Many hospitals that have failed to automate have seen their profits decline.
C. Unemployment insurance and taxes are paid also by corporations that are not
automating.
D. Many workers who have already lost their jobs to automated equipment have been
unable to find new employment.
E. The initial investment in machinery for automated equipment is often greater than
the short-term savings in labor costs.
The best answer is D. The threat envisioned by the author to the economic survival of
workers displaced by automation will be serious only if they cannot find new jobs.
Choice A states that there are already workers that cannot find employment, and so
strengthens the authors argument.

Many people claim that advertisements of alcoholic drinks influence young people to
start drinking socially. In Iceland, however, where there has been a ban on the
advertisement of alcoholic beverages since 1982, drinking is at least as prevalent
among young people as it is in countries that do not have a similar ban.
Which of the following statements draws the most reliable conclusion from the
information above?
A. Advertising does not play a role in causing young people to start or continue
drinking.
B. Advertisements of alcoholic drinks cannot be the only factor that affects the
prevalence of drinking among young people.
C. Banning advertisements of alcoholic drinks does not reduce the consumption of
alcoholic drinks.
D. More youths drink if they are not exposed to advertisements of alcoholic drinks
than if they are.
E. Most youths who drank in 1982 did not stop when the ban on advertisements of
alcoholic drinks was implemented.
The best answer is B. If advertisements of alcoholic drinks were the only factor that
affected young peoples drinking habits, there would be a difference in the prevalence
of drinking between countries that ban such advertising and those that do not.
According to the passage, there is no difference, so advertisements of alcoholic drinks
cannot be the only factor.

The Burmese ferret badger burrows its home in the ground. Based on the fact that
different local populations of Burmese ferret badgers of the same species dig homes
of different styles, zoologists have concluded that the badgers building styles are a
culturally acquired, rather than a genetically transmitted, trait.
Which of the following, if true, would most strengthen the conclusion drawn by the
zoologists?
A. There are more common characteristics than there are differences among the
Burmese ferret badgers styles of the local Burmese ferret badger population that has
been studied most extensively.
B. Young Burmese ferret badgers are inept at digging burrows and apparently spend
years watching their elders before becoming accomplished in the local style.
C. The homes of one species of badger lack the characteristics of the homes of most
other species of badger.
D. Burmese ferret badgers are found only in Burma and India, where local
populations of the badgers apparently seldom have contact with one another.
E. It is well known that the squeals of some badgers are learned rather than
transmitted genetically.
The best answer is B. The information in choice B says that young Burmese ferret
badgers progress slowly towards mastery of a burrow-digging style. This suggests
that the skill is one they must learn, rather than one whose transmission is wholly
genetic. Choice B also suggests a means of cultural transmission, namely,
observation of older badgers techniques.

The cost of manufacturing sports shoes in Macao is 25% less than the cost of
manufacturing them in the Philippines. Even after transportation fees and tariff
charges are added, it is still cheaper for a company to import sport shoes from Macao
to the Philippines than to manufacture sports shoes in the Philippines.
The statements above, if true, best support which of the following assertions?
A. Labor costs in Macao are 25% below those in the Philippines.
B. Importing sports shoes from Macao to the Philippines will eliminate 25% of the
manufacturing jobs in the Philippines.
C. The tariff on sports shoes imported from Macao to the Philippines is less than 25%
of the cost of manufacturing sports shoes in the Philippines.
D. The fee for transporting a pair of sports shoes from Macao to the Philippines is
more than 25% of cost of manufacturing the shoes in Macao.
E. It takes 25% less time to manufacture a pair of sports shoes in Macao than it does
in the Philippines.
The best answer is C. If the tariff on importing sport shoes from Macao to the
Philippines were as high as 25% or more of the cost of producing sports shoes in the
Philippines, then, contrary to what the passage says, the cost of importing sports shoes
would be equal to or more than the cost of producing sports shoes in the Philippines.
Thus, the tariff cannot be that high.

Opponents of laws that require motorcycle riders to wear helmets argue that in a free
society people have the right to take risks as long as the people do not harm others as
a result of taking the risks. As a result, they conclude that it should be each persons
decision whether or not to wear a helmet.
Which of the following, if true, seriously weakens the conclusion drawn above?
A. Many new motorcycles are built with safety features that made them less likely to
be involved in an accident.
B. Motorcycle insurance rates for all motorcycle owners are higher because of the
need to pay for the increased injuries or deaths of people not wearing helmets.
C. Participants in certain extreme sports are required to wear helmets.
D. The rate of automobile fatalities in states that do not have mandatory helmet laws
is greater than the rate of fatalities in states that do have such laws.
E. In motorcycle accidents, a greater number of passengers who do not wear helmets
are injured than are passengers who do wear helmets.
The best answer is B. The principle that people are entitled to risk injury provided
they do not thereby harm others fails to justify the individuals right to decide not to
wear a helmet if it can be shown that it does harm others by raising insurance rates.

A drug that is highly effective in treating certain types of cancerous growth can, at
present, be obtained only from the cartilage of a particular sub-species of shark, a subspecies which is quite rare in the wild. One must kill 50 sharks to make one pound of
the drug. If follows, therefore, that continued production of the drug must inevitably
lead to the extinction of this sub-species of shark.
Which of the following, if true, most seriously weakens the argument above?
A. The drug made from the shark cartilage is dispensed to doctors from a central
authority.
B. The drug made from the shark cartilage is expensive to produce.
C. Other organs of the shark can be utilized to produce different drugs.
D. The sub-species of shark will reproduce in captivity under the proper conditions.
E. This sub-species of shark generally lives in largely inaccessible waters.
The best answer is D. If the shark can be successfully bred in captivity, it is possible
to continue production of the drug without threatening the shark with extinction.

Harpers Pencils manufactures and sells the same pencils as Johnsons Supply.
Employee wages account for forty percent of the cost of manufacturing pencils at
both factories. Harpers is seeking a competitive edge over Johnsons supply.
Therefore, to promote this end, Harpers should lower employee wages.
Which of the following, if true, seriously weakens the argument above?
A. Because they make a small number of specialty artists pencils, pencil
manufacturers cannot receive volume discounts on raw materials.
B. Lowering wages would reduce the quality of employee work and this reduced
quality would lead to lowered sales.
C. Harpers Pencils has taken away twenty percent of Johnsons Supply business over
the last year.
D. Johnsons Supply pays its employees, on average, ten percent more than does
Harpers Pencils.
E. Many people who work for manufacturing plants live in areas in which the
manufacturing plant they work for is the only industry.
The best answer is B. The effect of lowering wages is to reduce quality sufficiently to
reduce sales. This is a good reason to doubt that wage cuts would give Johnson any
competitive edge.

In some cities, many potters have been winning acclaim as artists. But since pottery
must be useful, potters must exercise their craft with an eye to the practical utility of
their product. For this reason, pottery is not an art.
Which of the following, is an assumption that supports drawing the conclusion above
from the reason given for that conclusion?
A. Some plates, bowl and vessels are made to be placed in museums where they will
not be used by anyone.
B. Some potters are more concerned than others with the practical utility of the
products they produce.
C. Potters should be more concerned with the practical utility of their products than
they currently are.
D. Artists are not concerned with the monetary value of their products.
E. An object is not an art object if its maker pays attention to the objects practical
utility.
The best answer is E. The argument concludes that pottery is not an art because
potters must consider the practical utility of their product. If it is true that an object is
not a work of art if its maker pays attention to the objects practical utility, the
conclusion is supported.

Activity levels of the green iguana vary in cycles that are repeated every 24 hours. It
is logical to assume that alteration in the intensity of incident light is the stimulus that

controls these daily biological rhythms. But there is much evidence to contradict this
hypothesis.
Which of the following, if known, is evidence that contradicts the hypothesis stated
above?
A. The temperature of the green iguana varies throughout the day, with the maximum
occurring in the late afternoon and the minimum in the morning.
B. While some animals, such as the rabbit, are much more active during the day,
others, such as moles, show greater activity at night.
C. When animals are transported from one time zone to another, their daily biological
rhythms adjust in a matter of days to the periods of sunlight and darkness in the new
zone.
D. Other types of iguanas display similar activity cycles even though they live in very
different climates.
E. Even when exposed to constant light intensity around the clock, some iguanas
display rates of activity that are much greater during daylight hours than at night.
The best answer is E. If iguanas that are exposed to light 24 hours a day display a
similar pattern of activity as those exposed to natural light and dark cycles this
constitutes evidence again the hypothesis that it is the alterations in light that control
biological cycles.

Researchers recently discovered that lipstick can become contaminated with bacteria
that causes mononucleosis. They found that contamination usually occurs after the

lipstick has been used for six months. For that reason, women should dispose of any
lipstick after six months of use.
Which of the following, if true, would most weaken the conclusion above?
A. The researchers could not discover why lipstick contamination usually occurred
only after six months of use.
B. The researchers failed to investigate contamination of lipstick by viruses, yeasts,
and other pathogenic microorganisms.
C. The researchers found that among women who used lipstick contaminated with
bacteria that causes mononucleosis, the incidence of these diseases was no higher than
among people who used uncontaminated lipsticks.
D. The researchers found that people who wiped their lipsticks clean after each use
were as likely to have contaminated lipsticks as were people who never wiped their
lipsticks.
E. The researchers found that, after six weeks of use, greater length of use of a
lipstick did not correlate with a higher number of bacteria being present.
The best answer is C. According to choice C, using a contaminated lipstick does not
increase the incidence of infection, so the recommendation to replace a lipstick before
it becomes contaminated is greatly undermined.

Unlike the wholesale price of uncut diamonds, the wholesale price of other uncut
gemstones has fallen considerably in the last year. Thus, although the retail price of

jewelry made with gemstones other than diamonds has not yet fallen, it will inevitably
fall.
Which of the following, if true, most seriously weakens the argument above?
A. The cost of processing uncut gemstones has increased during the last year.
B. The wholesale price of uncut diamonds is typically higher than that of the same
volume of other uncut gemstones.
C. The operating costs of the average retail jewelry store have remained constant
during the last year.
D. The cost of mining gemstones other than diamonds has increased in the last year.
E. Changes in retail prices always lag behind changes in wholesale prices.
The best answer is A. The argument concludes that declining wholesale prices for
other gemstones will produce declining retail prices for jewelry made with gemstones
other than diamonds. Choice A weakens the argument by pointing to higher
processing costs for other gemstone, which could offset lower wholesale prices.
A company has initiated a health program for its employees that enables the worker,
free of charge, to receive a monthly massage treatment, consult with a dietician and
attend lectures in healthy living. These programs increase worker productivity and
absenteeism for employee health care. Therefore, these programs provide as much
benefit to the company as they do to the employees.
Which of the following, if true, most significantly strengthens the conclusion above?
A. Health programs are often the most popular services offered to employees.
B. Studies have show that such health programs are not effective for many people.
C. Regular massage treatments and dietary counseling reduce peoples risk of
catching a flu or a cold and provides them with increased energy.
D. Dizziness sometimes results from beginning a course of massage treatments.
E. Employee assistance programs require companies to hire people to supervise the
various programs offered.
The best answer is C. The conclusion is that the programs benefit both companies
and employees. For companies, reducing employees risk of flu or cold is likely to
reduce absenteeism, and increasing employee energy is likely to increase worker
productivity. For employees, the benefits are self-evident.
The next two questions are based on the following.

Interviewer: An alarming statistic reported in the Hobern Medical Journal is that 90


percent of the people in this country now report that they know someone who has
heart disease.
Dr. Summer: But an expected level of heart disease is 5 percent, or in other words, 1
out of every 20 people. So at any given time if a person knows approximately 50
people, 1 or more will very likely suffer from heart disease.
1st question: Dr. Summers argument is structured to lead to which of the following
conclusion?
A. The fact that 90 percent of the people know someone who suffers from heart
disease is not an indication that heart disease is abnormally high.
B. The current level of heart disease is not moderate.
C. If at least 5 percent of the population suffered from heart disease, the result of
questioning a representative group of people cannot be the percentage the interviewer
cites.
D. It is unlikely that the people whose statements the interviewer cites are giving
accurate reports.
E. If a person with heart disease is given as a certain percent, the actual percentage of
those with heart disease is even higher.
The best answer is A. Dr. Summers argument is essentially that, even if the facts are
as the interviewer presents them, they are not in and of themselves a cause for alarm.
Even circumstances reassuringly normal and unremarkable imply the sort of fact the
interviewer cites. Thus, that fact does not indicate that the rate of heart disease has
increased above normal.
Dr, Summers argument relies on the assumption that
A. normal levels of heart disease are rarely exceeded
B. heart disease is not normally concentrated in geographically isolated segments of
the population.
C. the number of people who each know someone who suffers from heart disease is
always higher than 90 percent of the population
D. the interviewer is not consciously distorting the statistics he presents
E. knowledge that a personal acquaintance has heart disease generates more fear of
getting heart disease than does knowledge of heart disease statistics.

The best answer is B. Dr. Summers argument assumes that people are generally
similar in how likely they are to have among their acquaintances people who have
heart disease. Since heavy concentrations of people with heart disease in
geographically isolated segments of the population would produce great differences in
that respect, Dr. Summers argument assumes few, if any, such concentrations.

Olympia buys free ride coupons from people who are awarded the coupons by
Plymouth Bus Lines for frequent travel on their bused. The coupons are sold to
people who pay less for the coupons than they would pay by purchasing tickets from
Plymouth Bus Lines. This marketing of coupons results in lost revenue for Plymouth
Bus Lines.
To discourage the buying and selling of free ride coupons, it would be best for
Plymouth Bus lines to restrict the
A. number of coupons that a person can be awarded in a particular year
B. number of routes on which travelers can use the coupons
C. days that the coupons can be used to Monday through Friday
D. amount of time that the coupons can be used after they are issued
E. use of the coupons to those who were awarded the coupons and members of their
immediate families
The best answer is E. Restricting the use of coupons to immediate family would
make the coupons valueless for anyone else, so that they could no longer be marketed.

The next two questions are based on the following:


The fewer restrictions there are on the advertising of medical services, the more
doctors who advertise their services, and the doctors who advertise a specific service
usually charge less for that service than doctors who do not advertise. Therefore, if
the nation rescinds the restrictions restricting medical advertisements that do not
specify fee arrangements, overall consumer medical costs will be lower than if the
nation retains its current restrictions.

Which of the following, if true, seriously weakens the argument concerning overall
consumer medical costs?

A. The nation has recently removed some other restrictions that had limited the
advertising of medical services.
B. The nation is unlikely to remove all of the restrictions that apply solely to the
advertising of medical services
C. Doctors who do not advertise generally provide medical services of the same
quality as those provided by doctors who do advertise
D. Most doctors who now specify fee arrangements in their advertisements would
continue to do so even if the specification were not required.
E. Most doctors who advertise specific services do not lower their fees for those
services when they begin to advertise.
The best answer is E. If E is true, the doctors who begin advertising when the
restriction is removed might all be among those who do not lower their fees on
beginning to advertise, in which case no decrease in consumer medical costs will
occur. Therefore, E weakens the argument.

If the statements above are true, which of the following must be true?
A. Some doctors who now advertise will charge more for specific services if they do
not have to specify fee arrangements in the advertisements.
B. If the restrictions against advertisements that do not specify fee arrangements is
removed, more doctors will advertise their services.
C. More consumers will use medical services if there are fewer restrictions on the
advertising of medical services.

D. If more doctors advertise lower prices for specific services, some doctors who do
not advertise will also charge less than they currently charge for those services.
E. If only restrictions on the advertising of medical services were those that apply to
every type of advertising, most doctors would advertise their services.
The best answer is B. The supposition in B involves reducing by one the number of
restrictions on the advertising of medical services. Any such reduction will, if the

stated correlation exists, be accompanied by an increase in the number of doctors


advertising their services, as B predicts.

Tougher hiring standards have not been the primary cause of the present staffing
shortage of registered nurses in hospitals. The shortage of nurses is primarily caused
by the fact that in recent years nurses have not experienced any improvements in
working conditions and their salary increases have not kept pace with increases in the
salaries of other hospital staff.
Which of the following, if true, would most support the claims above?
A. Many nurses already in the profession would not have been hired under the new
hiring standards.
B. Today more nurses are entering the profession with higher educational
qualifications than in the past.
C. Some nurses have cited higher standards for hiring as a reason for the current
staffing shortage.
D. Many nurses have cited low pay and lack of professional autonomy as reasons for
their leaving the profession.
E. Many prospective nurses have cited the new hiring standards as a reason for not
entering the profession.
The best answer is D. Choice D provides corroborative evidence for the claim
that the shortage of nurses results from deficiencies in pay and working
conditions by suggesting that for many former nurses, poor pay and working
conditions were reasons for their quitting the profession.

When a particular country halted their general conscription, people tended to save
more of their money, but when the general conscription was reinstated, people tended
to spend more of their money. The perceived threat of war, therefore, decreases the
willingness of people to postpone consumption for the sake of saving money.
The arguments above assumes that
A. the perceived threat of war has increased over the years
B. most people supported the development the general conscription in this country

C. peoples perception of the threat of war depends on whether there is a limited or a


general conscription in the country
D. the people who saved the most money when conscription was limited were the
ones who supported such limitations
E. there are more consumer goods available when the country has general
conscription
The best answer is C. On the basis of an observed correlation between conscription
and peoples tendency to save money, the argument concludes that there is a causal
connection between a perception of threat of war and the tendency not to save. That
connection cannot be made unless one links the perception of threat to conscription.

With the proliferation of private companies that specialize in genetic research, it was
feared that they would impose silence on the results of in-house research. This
constraint, in turn, would slow the progress of genetic engineering to alleviate patient
suffering.
Which of the following, if true, most seriously weakens the prediction of scientific
secrecy above?

A. Genetic research funded by industry has reached some conclusions that are of
major scientific importance.
B. When the results of scientific research are kept secret, independent researcher
are unable to build on those results.

C. To enhance the companies standing in the community, the genetic companies


encourage employees to publish their results, especially results that are important.
D. Since the research priorities of companies that specialize in genetic research are
not the same as those of academic institutions, the financial support of research by
such companies distorts the research agenda
E. Companies that specialize in genetic research devote some of their research
resources to problems that are of fundamental scientific importance and that are
not expected to produce immediate practical applications.
The best answer is C. Choice C weakens the prediction of secrecy by establishing
that companies specializing in genetic research have a strong motive to encourage
their researchers to publicize results.

A researcher discovered that people who have low levels of vitamin B12 tend to score
much lower on tests of short and long term memory than do people with normal or
high levels of B12. The researcher concluded from this experiment that vitamin
B12protects against problems of short and long term memory.
The researchers conclusion depends on which of the following assumptions?
A. High levels of B12 protect against memory problems better than normal B12 levels
do.
B. Memory problems are similar to malnutrition in its effects on body systems.
C. People with high levels of B12 cannot develop memory problems.
D. There is no third factor involved which causes both memory problems and a
reduction in the levels of B12 in the body.
E. Memory improvement exercises are not as effective as is medical treatment.

The best answer is D. The researcher concludes from the association of low levels of
B12 and memory problems that, in effect, the former causes the latter. If, there is a
third factor involved which effects the two, the association mentioned does not
support the researchers conclusion.

Reaching a diagnosis on the basis of step-by-step plan to search for the cause of a
disease has been considered preferable to intuitive searching. However, a recent
study found that hospital department heads used intuition significantly more than did
most doctors with less seniority. This confirms the alternative view that intuition is
actually more effective than careful, methodical reasoning.
The conclusion above is based on which of the following assumptions?
A. Department heads are more effective at decision making than doctors with less
seniority.
B. Department heads have the ability to use either intuitive reasoning or methodical
reasoning in making decisions.
C. The diagnosis made by doctors with less seniority can be made as easily by using
methodical reasoning as by using intuitive reasoning.
D. The most prominent doctors use intuitive reasoning in making the majority of their
decisions.

E. Methodical, step-by-step reasoning is inappropriate for making many life or


death medical decisions.

The best answer is A. If department heads are not the more effective decision
makers, then the fact that they use intuition more often than doctors with less seniority
does not support the conclusion that intuition is more effective.

The imposition of quotas limiting imported wheat will not help large local farms. In
fact, the quotas will help small local farms flourish. Those small domestic farms will
take more business from the large farms than would have been taken by the foreign
farms in the absence of quotas.
Which of the following, if true, would cast the most serious doubt on the claim made
in the last sentence above?
A. Quality rather than price is a major factor in determining the type of wheat bought
by a particular client.
B. Foreign wheat growers have long produced grades of wheat comparable in
quality to the wheat grown locally.

C. Local quotas on imported goods have often induced other countries to impose
similar quotas on this countrys goods.
D. Small domestic farms consistently produce better grades of wheat than do the big
farms.
E. Small domestic farms produce low-volume, specialized types of wheat that are not
produced by the large local farms.
The best answer is D. Choice D tends to support the claim, since better quality wheat
should sell better than poorer quality wheat.

Many mass marketed white breads are fortified with vitamin supplements. Some of
these breads provide 100 percent of the daily requirement of vitamin A, riboflavin,
niacin, pyridoxine, folic acid, iron and zinc. Nevertheless, a well-balanced diet,
including a variety of foods, is a better source of those vitamins than are such fortified
breads alone.
A. Unprocessed cereals are naturally high in several of the vitamins that are usually
added to fortified breakfast cereals.
B. People who regularly eat fortified bread sometimes neglect to eat the foods in
which the vitamins occur naturally.

C. Foods often must be fortified with vitamin supplements because naturally


occurring vitamins are removed during processing.
D. In many foods, the natural combination of vitamins with other nutrients makes
those vitamins more accessible to the body than are vitamins added in vitamin
supplements.
E. Cereals containing vitamin supplements are no harder to digest than similar cereals
without added vitamins.
The best answer is D. By pointing out that, when occurring in natural combinations
with other nutrients, vitamins are more usable by the body than those same vitamins
when added as a supplement, choice A provides a reason to believe that a wellbalanced diet is a better source of vitamins than is fortified bread.

The technological conservatism of art supply manufacturers is a reflection of the


kinds of demands they are trying to meet. The only customers who are seriously
interested in purchasing new products are professional artists. Therefore, innovation
in art supply technology is limited by what art critics and gallery owners accept as a
proper medium of expression for artists.
Which of the following is an assumption made in drawing the conclusion above?
A. The market for cheap traditional art supplies cannot expand unless the market
for new art products expands.

B. New art products are likely to be improved more as a result of technological


innovations developed in small workshops than as a result of technical
innovations developed in major manufacturing concerns.
C. Professional artists do not generate a strong demand for innovations that fall
outside what is officially recognized as a standard medium for purposes of artistic
creations.
D. The technological conservatism of art supply manufacturers results primarily from
their desire to manufacture a product that can be sold without being altered to suit
different national markets.

E. The critics and gallery owners who set standards for high-quality art do not keep
themselves informed about innovations in art supplies.
The best answer is C. If professional artists, the only customers interested in
innovation, created a strong demand for innovations for purposes other than what is
officially recognized as a standard medium for purposes of artistic creations, then the
conclusion would not follow. Therefore, choice C, which states that professional
artists generate no such demand is assumed and is the best answer.

Although Binofram has been proven to alleviate certain skin rashes associated with
some medical conditions, many physicians no longer routinely prescribe this drug for
this purpose. A rash can stimulate the activity of the bodys T-cells and also inhibit
growth of some strains of virus.
If the statements above are true, which of the following conclusion is most strongly
supported by them?
A. Binofram, an effective rash medication, alleviates the itching and discomfort of
many illnesses.
B. Binofram can prolong a patients illness by eliminating certain skin rashes which
can be helpful in fighting some diseases.
C. Binofram inhibits the growth of the bodys T-cells, which are necessary for fighting
some illnesses.

D. The more T-cells a patients body produces, the less severe the patients illness
will be.
E. The focus of modern medicine is on inhibiting the growth of harmful viruses
within the body
The best answer is B. By stimulating T-cells and inhibiting the growth of certain
viruses, rashes can aid the body in fighting infection. However, Binofram can
eliminate the rash. Thus, as choice B states, Binofram can prolong a patients illness.

Up to here used for gmat real test


Besides the manager and owner, Antonios Pizza is staffed by 15 high school students
who are paid minimum wage. The government passed a statute two month ago
raising minimum wage by 9%. Though this led to a considerable increase in costs to
Antonios Pizza, profits have increased markedly.
Which of the following, if true, most helps to resolve the apparent paradox?
A. Over half of Antonios Pizzas operating costs consist of payroll expenditures; yet
only a small percentage of those expenditures go to pay the salaries of those that
work in the kitchen.
B. Antonios Pizzas customer base is made up primarily of people who earn, or who
depend on the earnings of others who earn the minimum wage.
C. The operating costs, other than wages, increased substantially after the increase in
the minimum wage rate went into effect.
D. When the increase in the minimum wage rate went into effect, Antonios Pizza
also raised the mangers wage rate.

E. The majority of Antonios Pizza employees work as cashiers, and most cashiers
are paid the minimum wage.
The best answer is B. By showing how the wage increase might have led to an
increase in Antonios Pizzas sales, choice B helps resolve this question.

Before hiring staff that deals directly with food, employees must, by law, have a chest
x-ray. Sometimes the results of the x-ray are inconclusive, though this is no reflection
on the examinee. Rather, such a result means that the test has failed to show whether
the examinee was healthy or not. Nevertheless, employers will sometimes refuse to
hire a job applicant because of an inconclusive chest x-ray result.
Which of the following conclusions can most properly be drawn from the information
above?

A. Most examinees with inconclusive chest x-ray results are in fact unhealthy.
B. Chest x-rays should not be used by employers in the consideration of job
applicants.
C. An inconclusive chest x-ray result is sometimes unfairly held against the
examinee.
D. A chest x-rays indicating that an examinee is unhealthy can sometimes be
mistaken.
E. Some employers have refused to consider the results of chest x-rays when
evaluating job applicant.
The best answer is C. The passage indicates that an inconclusive chest x-ray tells
nothing about the person who has taken the test, and yet employers sometimes refuse
to hire someone whose results from such a test are inconclusive. Treating lack of
information as if it were unfavorable evidence about a person can reasonably be
considered unfair.

Summerhill Community College found that, because the current curriculum has little
direct relevance to skills required in order to succeed in the job market, they have
been attracting fewer and fewer new students. So to attract students to their college,
the board proposed a curriculum that emphasizes technology related to computers.
Which of the following, if true, provides the strongest reason to expect that the
proposed curriculum will be successful in attracting students?
1. Many technological principles can be applied to computers.
B. Knowledge of technology is becoming increasingly important in understanding
todays world.

C. Equipment that a large producer of computers has donated to the college could be
used in the proposed curriculum.
D. The number of students interested in technology today is much lower than the
number of students interested in technology 20 years ago.
E. In todays world, the production and application of computers is of major
importance in many places of business.

The best answer is E. For the proposed curriculum change to attract students to
classes, computers must have direct relevance in order to succeed in the job market.
Choice E provides evidence that this is so, and thus is the best answer.

In the 1980s, myopia was the most common eye problem in Ohio, followed
hyperopia, or farsightedness, and glaucoma, in that order. The incidence for both
hyperopia and glaucoma increases with age, but the incidence rate for myopia is the
same for people of all ages. The average age of the population of Ohio residents is
expected to increase between 1990 and 2010.
Which of the following conclusions can be most properly drawn about chronic
medical conditions in Ohio from the information given above?

A. Myopia will be more common than either hyperopia or glaucoma by 2010


B. Hyperopia will be the most common eye condition in 2010.
C. The average age of people suffering from myopia will increase between 1990 and
2010.
D. Fewer people will suffer from myopia in 2000 than suffered from it in 1980.
E. A majority of the population will suffer from at least one of the medical conditions
mentioned above by the year 2010.
The best answer is C. Given that the incidence rate for myopia is the same for people
of all ages, and that the average age of the population will increase, it follows that the
average age of people suffering from myopia will increase.

A New Mexico farm that grows hothouse tomatoes produces in 10 acres of space what
it takes 20 acres of field area to produce. Expenses, such as electricity, are high,
however, and the tomatoes harvested cost 60 % more than Florida field tomatoes.

Which of the following, if true, best supports a projection that the New Mexico
tomato farm will be profitable?
A. Once the operators of the facility are experienced, they will be able to cut
operating expenses by about 7%.
B. There is virtually no scope for any further reduction in the cost per pound for
Florida tomatoes.
C. Unlike washed field tomatoes, the hothouse tomatoes are untainted by any
pesticides or herbicides and thus will sell at very high prices to such customers as
health food stores and organic produce outlets.
D. Since tomatoes ship relatively well, the market for the hothouse tomatoes is not
more limited to the New Mexico area than the Florida tomatoes are to Florida.
E. A second hothouse facility is being built in Nevada, taking advantage of
inexpensive electricity and high vegetable prices.
The best answer is C. Choice C presents an advantage to the hothouse tomatoes - it
can be sold to certain customers who are prepared to pay very high prices for it. This
supports the projection that the facility will be profitable.

Last year the unemployment rate in Milton, a small city, was 3.5 percent, but for the
current year it has been 6 percent. We can conclude that inflation is on an upward
trend and the rate will be still higher next year.
Which of the following, if true, most seriously weakens the argument above?
A. The unemployment rates were computed on the basis of a representative
sample of economic data rather than all of the available data.

B. Last year, the closure of one of the largest plant in Milton brought unemployment
above its recent stable annual level of 3.5 percent until the plant reopened 4
months later.
C. Increases in the number of jobs available are tied to the level of unemployment,
and at an unemployment rate of 6 percent or above, these increases constitute a
force causing further unemployment.
D. The 3.5 percent unemployment rate of last year represented a ten-year low.
E. Government intervention cannot affect the rate of unemployment to any
significant degree.

The best answer is B. According to choice B, last years unemployment figure was an
anomaly, and unemployment has returned to its recent stable level. There is thus less
reason to conclude that unemployment will rise any further.

Poor management, not changes in the economy, is responsible for the low profits
reported in Aztec Inc., a subsidiary of Wilson Industries, since its new CEO took
control. Maya Inc., another subsidiary has had to function in the same economic
conditions, but while Aztecs profits have been falling, Mayas profits have been
rising.
Which of the following, if true, most seriously weakens the argument above?
A. R & D at Aztec Inc. is also progressing at a slower rate.
B. Whereas the Aztec plant is located in a small town, Maya is situated in the
industrial area of the largest city in the state.
C. Both companies have been functioning in a recessed economy.
D. The products that are produced by the Aztec subsidiary are different from those
that have always been manufactured by the Maya subsidiary.
E. Aztecs new CEO reorganized the breakdown of departments within the company
with the intention of reducing the number of redundant positions.
The best answer is D. The argument assumes that production in the two companies
would be affected in the same way by economic conditions. By pointing out that the
products manufactured in the two subsidiaries differ, choice D undermines this
assumption.

Caesarian sections are three times more common in the United States than they are in
the Netherlands. To support a conclusion that much of the variation is due to
unnecessary surgical procedures, which of the following must be true?
A. Maternity wards in American and Dutch hospitals examine the records of
every operation to determine whether the surgical procedure was necessary.
B. The variation in prevalence of caesarian sections is unrelated to factors that
influence the incidence of birth complications that necessitate the legitimate
use of caesarian sections.

C. There are several categories of surgical procedures not related to birth that are
often performed unnecessarily.

D. For certain surgical procedures, it is difficult to determine after the operation


whether the procedures were necessary or whether alternative treatment would
have succeeded.
E. With respect to how often they are performed unnecessarily, caesarian sections are
representative of surgical procedures in general.
The best answer is B. To establish that much of the variation is due to unnecessary
surgical procedures, it is necessary to eliminate the possibility that the difference
between the U. S. and the Netherlands reflects a variation in the incidence of birth
complications which legitimately necessitate the use of a caesarian section. Choice B,
if established would eliminate this possibility and is thus the best answer.

Archeologists have recently unearthed a 250-page bible commentary in Safed. The


document is undated but contains clues to when it was written. The first 130 pages
are written by one hand, while the rest is written in a distinctively different hand.
Since a letter in the handwriting identified as that of the second writer reports on an
earthquake that killed many people in Safed in 1759, the commentary was probably
produced around that time.
Which of the following, if true, most strongly supports the hypothesis that the
commentary was produced around 1759?
A. Other than this commentary, there are no know samples of the handwriting of the
first writer.
B. According to the account by the second writer, the earthquake caused the collapse
of over fifty structures.
C. A commentary like the one unearthed would usually take between three to nine
months to write.
D. There was only one earthquake in Safed in the 18th century.
E. It is highly unusual for such a document to be written by more than one hand.
The best answer is D. Choice D is the best answer because if there had been other
earthquakes in the relevant period, one of these, instead of the earthquake of 1759,
might have been alluded to in the letter.

Baltimore Sweets sell their popular Chocbloc in a distinctive square package.


Sunshine Chocolates have recently introduced a new chocolate bar sold in a similarly
shaped package. Baltimore Sweet believe it is likely that many customers intending to
buy their Chocbloc candy bar will mistakenly buy the Sunshine product instead.

However, Sunshine Chocolates claims that this is not so because their label on their,
but not on Chocbloc, is red.
Which of the following, if true, most undermines the response of Sunshine
Chocolates?
A. Red is the background color on the label of many of the chocolate bars produced
by Sunshine Chocolates.
B. When the chocolate bars are viewed side by side, the Sunshine Chocolates new
bar is perceptibly thicker than the Baltimore Sweets bar.
C. Sunshine Chocolates, unlike Baltimore Sweets, displays its packaging
prominently in advertisements.

D. It is common for occasional purchasers to buy a chocolate bar on the basis of a


general impression of the most obvious feature of the bar.
E. Many popular chocolate bars are sold in packaging of a standard shape.
The best answer is D. According to this choice, at least some of Baltimore Sweets
occasional customers are likely to overlook the difference in labels and buy Sunshine
Chocolates product instead of theirs, Sunshine Chocolates response to Baltimore
Sweets complaint is undermined.

When three Indian-owned trains purchased from Transcontinental Trains crashed


within a two month time period, the Indian parliament ordered the acquisition of
three new Transcontinental trains as replacements. This decision surprised many
because it is customary for users to shun a product after it is involved in
accidents.

Which of the following, if true, provides the best indication that the decision of the
Indian government was well supported?
(A) Although during the previous year only one Transcontinental train crashed,
competing manufacturers had a perfect safety record.
(B) The Transcontinental-built trains crashed due to sabotage, but because of the
excellent quality of the trains, fire was averted increasing the number of
survivors.
(C) The Indian Railway Commission issued new guidelines for trains in order to
standardize safety requirements governing inspections.

(D) Consumer advocates pressured two major railway companies into purchasing
safer trains so that the public would be safer.
(E) Many Transcontinental Train employees had to be replaced because they found
jobs with the competition.
The best answer is B. If the train crashes were caused by a factor that had nothing to
do with the train itself, such as sabotage, and the quality of the train decreased the
number of fatalities, then there is good reason to purchase additional trains from
Transcontinental.

Recently, a court ruled that current law allows tour operators that travel to potentially
dangerous locations, such as the Amazon, to reject a potential customers if there is a
50 percent chance that the traveler would contract malaria on the trip. The presiding
judge justified the ruling, saying that it protected both travelers and tour operators.
This use of his court ruling as part of the law could not be effective if which of the
following were true?
(A) The best interests of travelers often conflict with the interests of tour operators.
(B) There is currently no legally accepted method existing for calculating the risk
of contracting malaria as a result of being exposed to it in a particular location.

(C) Some trips might involve health risks other than the risk of malaria.
(D) Travelers who have a 50 percent chance of contracting malaria may be unaware
that their risk is so great.

(E) The number of people signing up to take trips with a tour company might decline
if the company, by screening applicants for risk of malaria, seemed to suggest that
the trip entailed high risk of getting the disease.
The best answer is B. The use of the court ruling as part of the law could not be
effective if there were no accepted way of determining who the people susceptible to
malaria were. Choice b states that there is no accepted method of calculating the risk
of contracting malaria.

Since the national postal service in country X was privatized six months ago, delays
in the delivery of mail have increased by 20 percent. To combat this problem, more
sorting and delivering staff must be hired in the busiest regions of the country.
Which of the following, if true, casts the most doubt on the effectiveness of the
solution proposed above?
(A) The major causes of delays in the nation's busiest regions are bad weather
and antiquated sorting equipment.

(B) Since privatization began, the number of employees has increased by 25 percent.
(C) Over 60 percent of the postal budget goes to pay salaries.
(D) After a small post sorting facility doubled its staff, the number of delays that were
reported decreased by 50 percent.
(E) Since privatization, the average length of delay in the nation's busiest regions has
doubled.
The best answer is A. The conclusion that postal service must add to their staff is
based on an assumption that the problem lies in that area. Choice A indicates that the
delay were due to two other factors bad weather and old equipment, therefore hiring
additional staff is unlikely to be an effective solution.

Employees that get a thorough medical examination twice a year take fewer sick days.
Even employees who get examined only once a year take less sick time than those
who do not get checked. Therefore, if companies instituted in-house medical
examination programs, the absentee rate in those companies would decrease
significantly.
Which of the following, if true, most seriously weakens the argument above?
(A) Employees who get medical check ups during working hours occasionally feel
ill for short periods of time after the examination.

(B) Employees who are frequently absent are the least likely to cooperate with a
corporate medical program.

(C) Employees who get a thorough medical examination once a week in their
company's medical program usually also get checked by their private doctor.
(D) Employees who get examined in their company's in-house medical program use
their working time no more productively than those who do not get examined.
(E) Employees who get medical examinations during working hours take slightly
longer lunch breaks than employees who do not get examinations.
The best answer is B. The argument states that absenteeism would decrease if an inhouse medical program were instituted. However, if those employees who are most
frequently absent are least likely to get examined, then the program is unlikely to
solve the problem of absenteeism.

In an attempt to discourage the widespread acceptance of clothing manufactured in


sweatshops, the council of a small town plans to ban the sale of sweatshop goods for
which substitutes manufactured under humane conditions exist. The council argues
that non-sweatshop apparel is morally preferable.
Which of the following, if true, indicates that the plan to ban the sale of sweatshop
goods is ill suited to the town council's moral objectives?

(A) Although clothing manufactured in sweatshops is now available, members of the


town council believe non-sweatshop apparel is better for workers in the clothing
manufacturing business.
(B) The clothing factory at which most of the townspeople are employed plans to
improve their working conditions.
(C) After other towns enacted similar bans on the sale of clothing manufactured in
sweatshops, the benefits to workers in the clothing manufacturing business were
not discernible for several years.
(D) Since most townspeople prefer clothing manufactured in sweatshops in many
instances, they are likely to purchase them in neighboring towns where such
goods are available for sale.
(E) Sweatshops sometimes produce items other than apparel.
The best answer is D. If the towns residents are likely to purchase the same items
from another town, then the town council's objectives of limiting purchases of
sweatshop manufactured clothing will not be met.

Holdens Ltd. two subsidiaries performed with remarkable consistency over the past
five years: in each of those years, Lexton has accounted for roughly 30 percent of
dollar sales and 60 percent of profits, and Stillmore for the balance.
Which of the following can properly be inferred regarding the past five years from the
statement above?
(A) Total dollar sales for each of the subsidiaries have remained roughly constant.
(B) Lexton has faced stiffer competition in its markets than has Stillmore.
(C) Stillmore has realized lower profits per dollar of sales than has Lexton.
(D) The product mix offered by each of the company's divisions has remained
unchanged.
(E) Highly profitable products accounted for a higher percentage of Stillmore's sales
than of those of Lexton.
The best answer is C. If Lexton has accounted for roughly 30 percent of dollar sales
and 60 percent of profits, then it has realized more profit per dollar of sales than
Stillmore. There are not enough facts to support the inferences reached in the other
answers.

Shereen cosmetics sometimes discounts the price of its premium eye shadows to
retailers for a promotion period when the product is advertised to consumers. Such
promotions often result in a dramatic increase in amount of premium eye shadows
sold by the manufacturers to retailers. Nevertheless, the manufacturers could often
make more profit by not holding the promotions.
Which of the following, if true, most strongly supports the claim above about the
manufacturers' profit?
(A) The amount of discount generally offered by manufacturers to retailers is
carefully calculated to represent the minimum needed to draw consumers'
attention to the eye shadows.
(B) For many consumer products the period of advertising discounted prices to
consumers is about a week, not sufficiently long for consumers to become
used to the sale price.

(C) For cosmetics that are not newly introduced, the purpose of such promotions is to
keep the products in the minds of consumers and to attract consumers who are
currently using competing products.
(D) During such a promotion retailers tend to accumulate in their warehouses
inventory bought at discount; they then sell much of it later at their regular price.
(E) If a manufacturer fails to offer such promotions but its competitor offers them,
that competitor will tend to attract consumers away from the manufacturer's
product.
The best answer is D. According to choice D, the promotion does not bring about
increased sales to the consumer, but a hoarding of the product by the retailer. Without
the promotion, retailers would eventually buy the same numbers of the product at its
full price, therefore, choice D supports the claim.

The next two questions are based on the following:

New home buyers are all financially protected against bankruptcy of the contractor
because of a law requiring the contractor to get bank backing to insure all individual
investments. An economist argues that this insurance is partly responsible for the high
rate of bankruptcies among contractors, since it removes from buyers any financial
incentive to find out the financial status of the contractor they are hiring. If buyers
were more selective, then contractors would need to be secure in order to compete for
buyers.
The economist's argument makes which of the following assumptions?
(A) Bankruptcy is caused when contractors default on loan repayments to their banks.
(B) A significant proportion of contractors use the services of sub-contractors to do
the bulk of the work.
(C) The more a home buyer has to invest in a house he or she is building, the more
careful he or she tends to be in selecting a contractor.
(D) The difference in the payment schedules to contractors is not a significant factor
in bankruptcies.
(E) People looking for a contractor to build their houses are able to determine which
contractors are secure against bankruptcy.
The best answer is E. The economists assumption is that what he is proposing can be
implemented. If those people looking for a contractor are unable to determine which
ones are secure against bankruptcy, then buyers do not have the option of being more
selective in their choice of contractor.

Which of the following, if true, most seriously weakens the economist's argument?
(A) Before the law requiring the contractor to get bank backing to insure all individual
investments was passed, there was a lower rate of bankruptcy than there is now.

(B) When the law did not insure buyers against the bankruptcies of building
contractors, frequent bankruptcies occurred as result of depositors' fears of
investing money in a newly built house.
(C) Surveys show that a significant proportion of new home buyers are aware that
their investments are protected by law.
(D) There is an upper limit on the amount of an individual's investment for which a
contractor is required by law to get bank backing, but very few individuals'
investments exceed this limit.
(E) The security of a contractor against bankruptcy depends on the percentage of its
assets reinvested into the business.
The best answer is B. The argument that bank backing is partially responsible for the
high rate of bank failures would be weakened if the bank backing to insure all
individual investments also prevented certain contractor bankruptcies. Choice B
suggests that it does prevent some bankruptcies and is thus the best answer.

Organic farmers are looking for non-chemical methods to rid their crops of pests.
Giving crops a slight electric shock has no ill effect on crops but rids them of
caterpillars. This method should be employed by organic farmers all over the
country.

Which of the following, if true, most strongly supports the view that it would not be
advisable to try to eradicate agricultural pests with the method mentioned above?
(A) Most species of caterpillar are subject to some natural predators.
(B) Many agricultural pests do not go through a caterpillar stage.
(C) This method also kills insects that are agriculturally beneficial.
(D) Since caterpillars of different species appear at different times of the year, several
shocks would be necessary.
(E) No large-scale production facilities exist today for the product that electrifies
crops.

The best answer is C. If electrifying crops kills beneficial insects along with the
pests, it may do as much or more harm than good.
A study of business partnerships in which one partners management style differs
from that of the other partner reveals that such partnerships are more likely be fraught
with problems and to ultimately end in dissolution. Thus, mismatched management
styles can seriously jeopardize a business partnership.
Which of the following, if true, most seriously weakens the argument above?
(A) Business partnerships in which both partners have the same management
style also occasionally have disagreements that can jeopardize the partnership.

(B) The management style of individuals tends to vary from year to year.
(C) The individuals who have management style that differ significantly from those of
their partners tend to argue little with spouses at home.
(D) Partners who are considering dissolution have been found to express hostility by
adopting a different management styles from that of their partners.
(E) According to a recent study, most peoples management styles can be controlled
and modified easily.
The best answer is D. If partners who are considering dissolution have been found to
express hostility by adopting a different management style from that of their partners,
then the different management styles are the result and not the cause of the
discordance between the partners.
A report on the use of monosodium glutamate in food concluded, Most
Americans who consume monosodium glutamate regularly are not being harmed.
Critics of the report insist the conclusion be changed to, Most Americans who
consume monosodium glutamate do not show visible symptoms of harm by the
substance, such as abnormal rashes or slower rates of metabolism.

Which of the following, if true, provides the best logical justification for the critics
insistence that the reports conclusion be changed?
(A) Some Americans who consume monosodium glutamate regularly are being
harmed by the substance.
(B) Monosodium glutamate could be causing long term damage for which
symptoms have not yet become visible.

(C) The report does not compare harm caused to Americans who consume
monosodium glutamate with harm caused to people in other countries.
(D) Monosodium glutamate has been added to a growing number of commonly
consumed foods during the past fifteen years.

(E) The severity of damage by monosodium glutamate differs from person to person.
The best answer is B. If there is a possibility that monosodium glutamate will
cause long-term harm, then the absence of symptoms cannot be taken as proof
that Americans who consume monosodium glutamate regularly are not being
harmed.

Until recently, most automobile manufacturers minimized the weight of their cars
to maximize mileage. The safest chassis were heavy, so few manufacturers
equipped their cars with the safest chassis. This year the chassis metal that has
sold best to automobile manufacturers has been the safest onea clear indication
that manufacturers are assigning a higher priority to safety than to mileage.

Which of the following, if true, most seriously weakens the argument above?
(A) Last year's best-selling chassis metal was not the safest chassis metal on the
market.
(B) No automobile manufacturer has announced that it would be making safe chassis
a higher priority this year.
(C) The price of gas was higher this year than it had been in most of the years when
the safest chassis sold poorly.
(D) Because of increases in the cost of materials, all car chassis were more expensive
to manufacture this year than in any previous year.

(E) Because of technological innovations, the safest chassis on the market this year
weighed less than some other chassis on the market.
The best answer is E. The conclusion is based on an assumption that the safest
chassis are still the heaviest ones, as they have been in the past. Choice E
proves this assumption to be erroneous.

The CEO of Blacks International has proposed replacing their Marmacil


semiconductor manufacturing equipment with Fasttech equipment since it costs 30
percent less to train new staff on the Fasttech equipment. Those opposed to the change
have pointed out the savings in training cost does not justify the change. Instead, they
suggested that the company hire only people who already know how to use the
Marmacil equipment.
Which of the following, if true, most seriously undermines the objection to the
replacement of Marmacil semiconductor manufacturing equipment with Fasttechs?

(A) Currently all employees in the company are required to attend workshops on how
to use Marmacil semiconductor manufacturing equipment in new applications.
(B) Once employees learn how to use semiconductor manufacturing equipment, they
tend to change employers more readily than before.
(C) Experienced users of Marmacil equipment command much higher salaries than do
prospective employees who have no experience in the use of such equipment.

(D) The average productivity of employees in the general manager's company is


below the average productivity of the employees of its competitors.
(E) The high costs of replacement parts make Marmacil computers more expensive to
maintain than Fasttech semiconductor manufacturing equipment.
The best answer is C. Those objecting to the change claim that the advantage of
Fasttechs relatively low cost can be mitigated by hiring experience users of Marmacil
semiconductor manufacturing equipment. However, if such people command much
higher salaries as choice C states, doing so would not result in savings to the
company.

A pharmaceutical company developed a new diuretic reported to cause fewer side


effects than their old diuretic, which was still being manufactured. During the first
year that both were sold, the earlier medication far outsold the new one; the
manufacturer thus concluded that reducing side effects was not the customers' primary
consideration.
Which of the following, if true, would most seriously weaken the manufacturers
conclusion?
(A) Both hospitals and patients buy diuretics from this pharmaceutical company.
(B) Many customers consider older medications a better safety risk than new ones,
since more is usually known about the safety of the earlier drugs.

(C) Many customers of this pharmaceutical company also bought medications from
companies who did not produce new diuretics reported to cause fewer side
effects.
(D) The newer diuretic can be used by all the patients who could use the earlier
diuretic.
(E) There was no significant difference in price between the newer diuretic and the
earlier diuretic.
The best answer is B. The manufactures conclusion was that reducing side effects
was not the customers' primary consideration. Choice B states that customers
consider older medication a better safety risk, so those customers bought the older
diuretic out of safety considerations.

Canadian wheat farmers produced so much wheat over the last season that wheat
prices plummeted. The government tried to boost wheat prices by offering farmers
who agreed not to harvest 20 percent of their wheat field compensation up to a
specified maximum per farm.
The Canadian government's program, if successful, will not be a net burden on the
budget. Which of the following, if true, is the best basis for an explanation of how this
could be so?
(A) If prices were allowed to remain low, the farms would be operating at a loss,
causing the government to lose tax revenue on farm profits.

(B) Wheat production in several countries declined the year that the compensation
program went into effect in Canada.
(C) The first year that the compensation program was in effect, wheat acreage in
Canada was 5% below its level in the base year for the program.
(D) The specified maximum per farm meant that for very large wheat farms the
compensation was less per acre for those acres than they were for smaller
farms.

(E) Farmers who wished to qualify for compensation program could not use the land
not harvested for wheat to grow another crop.
The best answer is A. The program will not be a net burden on the budget because
money spent on compensation can be made up by money collected through taxation
of the farms.

Eclampsia in pregnancy is especially prevalent among women who are seriously


malnourished. In order to achieve early detection of eclampsia in these
individuals, public health officials distributed pamphlets explaining the importance
of early detection of this potentially fatal disease.

Which of the following, if true, is the best criticism of the use of the pamphlet as a
method of achieving the public health officials goal?
(A) Many prenatal diseases produce symptoms that cannot be detected by the
patient.

(B) Once eclampsia has been detected, the effectiveness of treatment can vary from
person to person.
(C) The pamphlet was sent to all town residents, including those individuals who are
not pregnant.
(D) Eclampsia is much more common in very young and very old mothers, rather than
in those at the height of their productive years.
(E) Pregnant women who are seriously malnourished are often homeless and thus
unlikely to receive and read the pamphlet.
The best answer is E. The pamphlet is incapable of achieving the public health
officials goal if it does not reach its target audience.

Technological improvements have made electric cars far more cost-efficient in the last
decade. However, the threshold of economic viability for electric cars (that is, the cost
of running an electric car compared to a gasoline powered car) has remained
unchanged.
Which of the following, if true, does most to help explain why the increased costefficiency of electric cars has not decreased its threshold of economic viability?

(A) The cost of manufacturing gasoline-powered cars has fallen dramatically.


(B) The reduction in the cost of manufacturing electric cars has occurred despite
increased raw material costs for those cars.

(C) Technological changes have increased the efficiency of gasoline-powered cars.


(D) Most electricity is generated by coal-fired or nuclear, rather than oil-fired, power
plants.
(E) When the price of oil increases, reserves of oil not previously worth exploiting
become economically viable.
The best answer is C. Improvements in electric cars have been matched by
improvements in gasoline-powered cars so that their economic viability has not
improved.

Generally, marine biologists enter their field with the goal of doing important new
research. They expect their colleagues to have similar goals. Therefore, when
any marine biologist becomes a household name in the field of documentary
nature films, most other scientists conclude that this biologist should no longer be
regarded as a true colleague.

The explanation offered above for the low esteem in which marine biologists who
make documentary films are held by research scientists assumes that
(A) serious scientific research is not a solitary activity, but relies on active
cooperation among a group of colleagues
(B) research scientists tend not to regard as colleagues those scientist whose renown
they envy
(C) a scientist can become famous as a marine biologist who makes documentary
films without having completed any important research
(D) research scientists believe that those who are well known as marine biologists
who make documentary films are not motivated to do important new research
(E) no important new research can be accessible to or accurately assessed by those
who are not themselves scientists
The best answer is D. The conclusion that biologists who make documentaries
should no longer be regarded as true colleagues is based on the assumption that
such biologists will not longer do important new research.

New stores financed by investors have a much lower failure rate than stores financed
by other means. Source of financing, therefore, must be a more important causative
factor in the success of a new store than are such factors as the location of the store,
the quality of the staff, or the choice of merchandise.
Which of the following, if true, most seriously weakens the argument above?
(A) Investors tend to be more responsive than others to changes in a new stores
financial needs.
(B) The strategic planning of a new store is a less important factor in the long-term
success of the business than are the personal characteristics of the owner.
(C) More than half of all new stores close within three years.
(D) The management of new stores is generally less formal than the management
of ongoing stores.

(E) Investors base their decisions to fund new stores on such factors as the personal
characteristics of the owner, location of the store, and marketing goals.
The best answer is E. The argument is that source of financing must be a more
important causative factor in the success of a new store than other factors. Choice E
suggests that it is not the source of financing that makes the difference, rather that
investors are more likely to finance new stores in which the other factors - good
locations, good quality of staff etc. - are good.

The proportion of Afro-American students enrolled in Gilmore Community College


has increased over the past decades. This is partly shown by the fact that in 1965, only
6 percent of Afro-American between twenty and twenty-three in the town of Gilmore
were enrolled in college, while in 1997, 13 percent of the Afro-Americans between
twenty and twenty-three were enrolled in Gilmore Community College.
To evaluate the argument above, it would be most useful to compare 1965 and 1997
with regard to which of the following characteristics?
(A) The percentage of Afro-American students between twenty and twenty-three who
were not enrolled in Gilmore Community College
(B) The percentage of Afro-American students between twenty and twenty-thirty who
graduated from Gilmore Community College
(C) The percentage of Afro-American students who, after attending Gilmore
Community College, entered highly paid professions
(D) The non-Afro-American students between twenty and twenty-three who were
enrolled in Gilmore Community College
(E) The percentage of -Afro-American students who graduated from high school
The best answer is D. The argument presents a substantial increase in the proportion
of Afro-Americans between twenty and twenty-three who were enrolled in Gilmore
Community College as evidence that there was an increase in the proportion of
Gilmore Community College students who were Afro-Americans. This evidence
would lack force if a similar increase in college enrollment had occurred among
students who are not Afro-American.

It would cost Hamilton Inc. two million dollars to stop the leakage of minute
amounts of dangerous chemicals into its plant. In the year after completion of
those repairs, however, Hamilton Inc. would thereby avoid incurring three million
dollars worth of damages, since currently Hamilton Inc. pays that amount
annually in compensation for health problems said to be caused by the chemical
fumes.

Which of the following, if true, gives the strongest support to the argument above?
(A) Companies similar to Hamilton Inc. also pay compensation for health damages
caused by fumes.
(B) After leaky Hamilton Inc. equipment has been repaired, several years will elapse
before that the equipment begins to leak again.
(C) Hamilton Inc. would need to raise their prices to consumers if it were to
spend two million dollars in one year on repairs.

(D) The number of sick days Hamiltons employees take can vary widely from year to
year.
(E) Factory workers are more likely to be exposed to fumes while in Hamiltons plant,
but administrative staff files almost all of the claims for compensation said to be
caused by the fumes.

The best answer is B. If choice B is true, one can assume that for
several years Hamilton will save three million dollars paid in
compensation, and that strengthens the argument that it is worth
making the repairs.

Two experimental fields were each planted with wheat. Potash was added to the first
field but not to the second. The first field produced 151 bushels of wheat per acre and
the second field produced 110 bushels of wheat per acre. Since nothing else but water
was added to either field, the higher yields in the first field must been due to the
potash.
Which of the following, if true, most seriously weakens the argument above?
(A) A small amount of the potash leached into the second field.
(B) Wheat in a third experimental field, to which a high-nitrogen fertilizer was added,
but no potash, produced 130 bushels of wheat per acre.
(C) Four different types of wheat were grown in equal proportions in each of the
fields.
(D) Some weeds that compete with wheat cannot tolerate high amounts of potash in
the soil.
(E) The two experimental fields were located in different states.

The best answer is E. If the two fields were located in different


states, then there were other factors such as soil and weather that
could have caused the differences in crop yields.
Normally, increases in the price of a product decrease its sales except when the price
increase accompanies an improvement in the product. Art is unusual, however. Often
increases in the price of a particular artists paintings will result in increased sales,
even when the size, medium and style of the painting remains unchanged.

Which of the following, if true, does most to explain the anomaly described above?
(A) The choice of paintings on the market by various artists is extremely wide.
(B) Many consumers make decisions about which painting to purchase on the
basis of art reviews published by galleries.

(C) Consumers selecting a painting in a gallery often use the price charged as their
main guide to the paintings quality.
(D) Gallery owners can generally increase the sales of a beginner artists paintings
temporarily by introducing a price discount.
(E) Buyers who purchase paintings regularly generally have strong opinions about
which artists they prefer.

The best answer is C. If price charged is used as the main guide


to the paintings quality, then the more expensive it is, the higher
quality the buyer judges it to be. Since it is perceived to be of
higher quality, it is likely to sell well.

The recent decline in art prices has hurt many art investors. Last year, before the
decline began, the Hillhurst Historical Society added 3 valuable paintings and 2
sculptures to its holdings. The society, however, did not purchase the works of art
but received them as a gift. Therefore, the price decline will probably not affect
the society.

Which of the following, if true, casts most doubt on the conclusion above?
(A) The works of art that the society was given last year are ones that the society itself
uncovered when examining an abandoned historical house in town.
(B) The society usually receives more contributions of money than of art.
(C) Art prices in the U.S., where the society is based, are currently higher than in
Europe.
(D) Last year, the amount that the society allocated to pay for restorations included
money it expected to receive by selling some of the works of art.
(E) Last year, the society paid no taxes on art received as gifts, but instead paid fees to
compensate the local government for services provided.

The best answer is D. The conclusion states that the price decline
will probably not affect the society since it received the works of art
as a gift. However, the conclusion does not take into consideration
that the society had counted on a certain level of revenue from the
immediate sale of some of the art. Since it will likely be unable to
sell the pieces of art at the intended prices, the decline in price will
affect the society.
Plaster of Paris is used by orthopedic surgeons to make casts for people with broken
limbs. These casts are kept on for up to three months the maximum time needed for
a broken limb to heal. Orthomed has developed a new material that can be used in
place of plaster of Paris. Orthomeds salespeople claim that their product will improve
healing because Orthomed can hold up twice as long as the currently used plaster of
Paris does.
Which of the following statements, if true, would most seriously call into question the
claim made by Orthomeds salespeople?
(A) Most broken bones take about five weeks to heal.
(B) Most material for casts is purchased by hospitals and clinics rather than by
individual surgeons.
(C) The durability of the currently used plaster is more than sufficient to bind broken
limbs for three months.
(D) Neither Orthomed nor the currently used plaster of Paris stops the skin under the
cast from itching.
(E) Orthomeds durability on skin that has been coated with a special chemical
preparation is only half as good as the durability of the currently used plaster on
such coated skin.
The best answer is C. If the currently used plasters durability is more than sufficient
to bind broken limbs for three months then Orthomeds claim that their product will
improve healing because Orthomed can hold up twice as long has no basis.
Advertisement:
The best cranberries come from Alabama. The more cranberries from Alabama in a
blend of cranberry juice, the better the blend, and no company purchases more
cranberries from Alabama than Cranbies Cranberries, Inc. So it only stands to reason
that if you buy a jar of Cranbies cranberry juice, you are buying the best available
today.
The reasoning of the argument in the advertisement is flawed because it overlooks the
possibility that

(A) the equipment used by Cranbies to blend and package its juice is no different
from that used by most other cranberry juice producers
(B) not all of Cranbies competitors use cranberries from Alabama in the blends of
juice they sell
(C) Cranbies sells more cranberry juice than does any other company
(D) Cranbies juice is the most expensive cranberry juice available today
(E) the best juice made from one crop of cranberry is better than the best juice made
from a blend of cranberries.

The best answer is C. The reasoning of the argument in the


advertisement is flawed because it is based on an assumption that
Cranbies product contains more cranberries from Alabama than
any other brand. However, if the reason Cranbies buys more
cranberries from Alabama is because they make more juice
overall, there may be a smaller percentage of cranberries from
Alabama in the juice they market.
In order to increase revenues, Rent-a-Bike, the only bicycle rental agency in town,
plans to change their rental fees. Rather than charging $8.00 for the first two-hour
period, or part thereof, and $4.00 for each hour thereafter, Rent-a-Bike will charge
$16.00 for the first four-hour period, or part thereof, and $4.00 for each hour
thereafter.
Which of the following is a consideration that, if true, suggests that the plan will be
successful in increasing revenues?
(A) Very few people who rent bicycles at the hourly rental agency rent them for
more than two hours at a time.

(B) Over the past several years, the cost of operating the bicycle rental facilities has
been greater than the revenues it has received from them.
(C) Tourists generally rent bikes from agencies that rent by the day rather than by the
hour.
(D) A significant portion of the money spent to operate Rent-a-Bike is spent to
maintain the facilities rather than to pay the salaries of the personnel who rent out
the bicycles.
(E) The number of bicycles owned by Rent-a-Bike has recently been increased and
are therefore the bicycle are rarely all rented out at one time.

The best answer is A. Since most people that rent bicycles do so


for no more than two hours at a time, they pay only $8.00 in total.
If the suggested plan is implemented, all customers will pay
$16.00.
Since a sambhar, a large, horned Asiatic deer, with no horn is worthless to poachers,
the Wildlife Protection Committee plans to protect selected sambhars from being
killed by poachers by cutting off their horns.
The Wildlife Protection Committees plan assumes that

(A) poachers do not kill sambhars for any reason other than to
obtain their horn

(B) hornless sambhars pose less of a threat to humans, including


poachers, than do sambhars that have horns

(C) sambhars are the only animals poachers kill for their horns

(D) hornless sambhars can successfully defend their young


against non-human predators

(E) imposing more stringent penalties on poachers will not


decrease the number of sambhars killed by poachers

The best answer is A. According to the Wildlife Protection


Committee, cutting off the horns of sambhars will discourage
poachers from killing them. However, if poachers hunt sambhars
for their meat as well, cutting off their horns is unlikely to be a
successful deterrent.

Fewer students are studying chemistry now than previously. If the economy grows,
the demand for educated chemists will far outstrip the supply. Some companies have
already started to respond to this situation by finding better ways to keep their current
chemists.
Their concern is a sure indicator that the economy is growing.

(A) The fact that companies are making prudent preparations for a possible future
development does not mean that this development is already taking place.
(B) The fact that some companies now try harder to keep their chemists does not
mean that they used to be indifferent to the morale of their employees.
(C) The fact that demand will outstrip supply does not mean that there will be no
supply at all.

(D) The fact that the number of new students studying chemistry is declining does not
mean that the number of new chemistry students is lower than it has ever been.
(E) The fact that currently employed chemists have become more valuable to some
companies does not mean that those employees will do their jobs better than they
used to.

The best answer is A. The companies that employ chemists have


made a prediction about the future and have acted accordingly. It
does not follow that their prediction that the economy is growing
. has already become a reality

Under current federal law, employers are allowed to offer their employees free inhouse daycare as a tax-free benefit, but they can offer employees only up to $400 per
year as a tax-free benefit for their childrens enrollment in local day care facilities.
The government could significantly increase enrollment in local day care facilities by
raising the limit of this benefit to meet employees costs.
Which of the following, if true, indicates that there is a major flaw in the argument
above?
(A) local day care facilities are not always as fully outfitted as in-house day care
facilities.

(B) enrollment in local day care facilities is better for the child because he or she
becomes acquainted with neighborhood children with which the child will
eventually attend school.
(C) the in-house day care facilities offered by employers as tax-free benefits can be
worth as much as $2,800 per year.
(D) many employees are deterred by financial considerations from enrolling their
children in local day care facilities.
(E) because of last minute changes in working hours, it is often easier for employees
to have their children in an in-house day care than in one located close to home.

The best answer is D. The argument states that the government could significantly
increase enrollment in local day care facilities by raising the limit of this benefit to
meet employees costs. The faulty assumption is that financial considerations are the
only ones that influence parents decisions about day care placement.

Cucumber plants native to Ecuador produce, on average, 3 pounds of produce per


plant, per year; if Ecuadorian cucumber plants are crossbred with European varieties,
the crossbred plants can produce, on average, 4.7 pounds per year. An international
agency plans to increase the profitability of Ecuadors cucumber produce by
encouraging a hybrid strain of native Ecuadorian cucumber plants with European
strains.
Which of the following, if true, casts the most serious doubt on the viability of the
agency's plan?
(A) Not all European strains of cucumber can be successfully crossed with native
Ecuadorian cucumber plants.

(B) Many young Ecuadorians now regard farming as a low-status occupation because
it is less lucrative than other endeavors open to them.
(C) Ecuadors terrain is suitable for raising native strains of cucumber but does not
have the soil necessary to sustain the hybrid crop.
(D) Cabbage and tomato crops, not cucumber, make up the bulk of Ecuador 's
vegetable exports to Europe.
(E) Many European strains of cucumber produce yields exceeding 4.7 pounds per
plant per year.

The best answer is C. The agencys plan is based on the


assumption that it is possible to produce high yields in Ecuador. If
the soil is unsuitable for growing the hybrid cucumber, it will be
impossible to produce high yields then the plan may, therefore, not
.be viable

Working excessively long hours inevitably leads to raised adrenaline levels.


Marketing executives at Thornhill Industries, all suffer from chronically raised
adrenaline levels. A majority of the marketing executives works all through the night
at least once every two weeks. In other companies, marketing executives rarely work
more than 50 hours a week. Thornhill Industries gives regular bonuses only to the five

marketing executives of the eight in the company who work the greatest number of
hours per week.
Which of the following conclusions is most strongly supported by the statements
above?
(A) Marketing executives at Thornhill Industries work under conditions that are more
stressful than the conditions under which marketing executives at most other
companies work.
(B) Most of the employee bonuses given by Thornhill Industries are given to
marketing executives.

(C) At Thornhill Industries, raised adrenaline levels is more widespread among


marketing executives than among any other group of employees.
(D) No marketing executive at Thornhill Industries who works only 40 hours per
week suffers from overwork.
(E) Most of the marketing executives at Thornhill Industries who receive regular
bonuses have chronically raised adrenaline levels.

The best answer is E. If the executives who receive regular


bonuses are those who work the longest hours, and working
excessively long hours inevitably leads to raised adrenaline levels,
then one can conclude that the Thornhill executive who receive
.regular bonuses have raised adrenaline levels

Summer profits, the total sales recorded between June and


August, determine the economic success or failure of seasonal
businesses in Sweden. Sunglow, a retailer selling a sunscreen
line, is an excellent example. Sunglows summer profits, on
average, account for a two thirds of its yearly total receipts and
.about three-quarters of its yearly profits

If the statements above are true, which of the following must also
?be true about Sunglow on the basis of them

Its fixed expenses per sunscreen sold are higher during the (A)
.summer season than for any other three-month period

It makes more profit during the fall and spring quarters (B)
.combined than during the summer season
Its per-product retail price is lower, on average, during the (C)
.summer season than during any other three-month period

It makes less profit, on average, for a given dollar amount of (D)


sales from September to April combined than during the
.summer season

The per-product price it pays to wholesalers is higher, on (E)


average, during the summer season than from September to
.April
The best answer is D. In order for the statement that Sunglows
summer sales count for two thirds of its yearly total receipts and
three quarters of its yearly profits to be true, it must make less
.profit per sale from September to April

Opthamologists can determine important information about blood


pressure level while performing an eye examination. Doctors have
found that high blood pressure levels can precede a cardiac arrest.
One can conclude that doctors will be able to predict with greater
accuracy impending heart attacks based on information from
.opthamologists

Which of the following, if true, most seriously weakens the


argument above
Eye examinations provide more detailed information today than (A)
.they did 35 years ago

Doctors can establish that changes in eye pressure directly (B)


.affect the cardiovascular system

Evidence other than eye pressure has previously enabled (C)


doctors to predict impending heart attacks that are predictable
.on the basis of information from opthamologists

Doctors have not determined why changes in pressure on the (D)


.eye fluctuate
It has been established that predictable pressure patterns yield (E)
.predictable pattern of medical symptoms

The best answer is C. The argument establishes that newly


available information about eye pressure can be used to predict
impending heart attacks. The conclusion is that doctors will be
able to use this information to predict with greater accuracy
impending heart attacks. Choice C states that the information
opthamologists provide can be obtained from other sources, so
there may be no improvement in doctors ability to predict
.earthquakes

Over the last fifteen year, reports indicate that coffee consumption in Canada has
increased by 40 percent. But over that same period, supermarkets, where the vast
majority of Canadian consumer purchase the coffee they consume at home, reported
only a 12 percent increase is coffee sales.
Which of the following statements about the last fifteen years in Canada is best
supported by the statements above?
(A) Spending on coffee increased at restaurants and coffee houses that prepare coffee
to be consumed away from home.
(B) Coffee prices at supermarkets rose dramatically compared to prices at popular
coffee house chains.
(C) Coffee prices at supermarkets neither rose nor dropped relative to coffee prices at
other coffee retailers.
(C) Former customers indicated that they had often been frustrated trying to locate
individual items in the supermarkets coffee merchandise displays.
(E) The number of people who ate at restaurants decreased relative to the number of
people who ordered take-out coffee from restaurants.
The best answer is A. If the consumption of coffee has increased, and
supermarket coffee sales have not increased to the same degree, consumers
must be consuming coffee elsewhere. It states in the question that the vast
majority of Canadian consumer purchase coffee for home consumption in
supermarkets, so the increase must be accounted for by an increase in coffee
consumed outside the home, such as in restaurants and coffee houses.

Since last year, when our hardware store enjoyed its highest sales figures ever,
customers have increasingly turned a competitor located 10 blocks away. We plan
to reverse this trend by reorganizing the electrical, tool and car part sections of
our store, simplifying our displays, and making individual items easier to find.
Which of me following, if true, most strengthens the viability of the hardware
store's plan?

(A) Many hardware stores have increased their sales by placing small but expensive
items near cash registers, where customers waiting to pay are tempted to buy
them on impulse.
(B) Some consumers who shop at the competitor consider it to be a good source of
information about home improvement.
(C) Former customers indicated that they had often been frustrated trying to locate
individual items in the hardware store's merchandise displays.

(D) Hardware store trends have changed since last year due to the popularity of do-ityourself television shows. Consumers are now spending less on ready-to-use item
and more on raw materials.
(E) Since last year, there has been a sharp rise in the number of mail-order merchants
who sell hardware goods.
The best answer is C. The stores plan to reorganize certain departments,
simplifying displays, and make items easier to find is a viable solution if former
customers have indicated that this was why they had stopped shopping at their
store.

Several tour operators have recently intensified their competition for business by
offering weekend deals that, if purchased 2 weeks in advance, cost as much as 35
percent less than the lowest-priced discount tickets, which require a 3 week
advance purchase. The tour operators long-term gains from tickets bought under
this new plan will be significant because, unlike earlier discount tickets,

these are not refundable.


The author assumes which of the following about discounted weekend deals in
predicting long-term gains in revenue?
(A) More discounted, 2-week advance purchase deals are purchased than are actually
used.
(B) Tickets requiring 3-week advance purchase are not profitable for tour operators.
(C) Few travelers with small children have taken advantage of 3-week advance
purchase tickets.
(D) Tour operators will have to discontinue offering 3-week advance purchase tickets
when they begin offering 2-week advance purchase tickets.
(E) The majority of the 2-week advance purchase tickets offered by a given tour
operator will be sold to passengers who have not previously taken tour with that
operator.
The best answer is A. Although the 3-week advance tickets cost more and can,
therefore, potentially create more revenue, the author assumes that this will be
made up for by the fact that the 2-week tickets cannot be returned.

A small furniture store uses a software program called Blaze to do its basic
bookkeeping. There is good reason to believe that the accounting it provides is
inaccurate. Several times when the accounts were carefully calculated by a

bookkeeper using an adding machine, the resulting total differed from the total the
computer gave.
The furniture store managers reasoning relies on which of the following
assumptions?
(A) The criteria that the computer uses in determining what items are included in the
accounting of a particular month differ from the criteria the bookkeeper uses.
(B) The inaccuracy of the computer's accounting does not result from a
malfunction of the computer itself.

(C) It would be possible to modify the computer so that it calculated the stores
accounts more accurately.
(D) A careful calculation by the bookkeeper using an adding machine is unlikely to be
less accurate than the computer's accounting.
(E) The accuracy of the computer's accounting is not dependent on the volume of
sales in a particular month.
The best answer is D. In order for the conclusion - that the accounting the
computer provides is inaccurate to be true, one must assume that the
calculation one checks it by, namely the bookkeepers manual accounting, is
accurate.

People who go year after year without ever getting the common cold tend to be
sociable adults who have a lot of friends. These individuals tend to get together
with friends at least once a week, so sociable adults can improve their chances of
living a long life by making sure they get together socially with friends at least
once a week.

A flaw in the argument above is that it


(A) gives reasons for the truth of its conclusion that presuppose the truth of that
conclusion
(B) proceeds as though a condition that by itself is enough to guarantee a certain
result must always be present for that result to be achieved
(C) assumes without proof that two phenomena that occur together share an
underlying cause
(D) concludes that one phenomenon is the cause of another when at most what has
been established is an association between them
(E) fails to recognize that a tendency widely shared by a subgroup within a given
population will not necessarily be widely shared by that population as a whole

The best answer is D. It is impossible to determine from the information supplies


above whether sociability protects against the common cold, whether the type of
person who does not catch the common cold tends to be more popular, or whether
there is a third factor which affects both health and popularity.

In the county of Templeton, the average age of cell phones that are still in use has
historically been eighteen months years, but now it is nearly two years. Cell phone
manufacturers claim that the current poor economy has forced people to put off
buying new cell phones, and thus when the economy improves, the average age of cell
phones will return to former levels.
Which of the following, if true, most seriously calls into question the cell phone
manufacturers' prediction?

(A) Fewer cell phone per year are now being marketed in Templeton than last year.
(B) When the threat of job loss is particularly strong, people are reluctant to commit
themselves to expensive purchases.
(C) The older a cell phone is, the greater the monthly insurance payments are.
(D) The cell phones now marketed in Templeton cost less than those that were
marketed last year.
(E) Most people in Templeton now believe that replacing an old cell phone with a new
one sooner than is necessary is pretentious.
The best answer is E. The prediction that the average age of cell phones will
return to former levels is based on an assumption that economic considerations
have changed the average length of time a Templeton consumer keeps a cell
phone. Choice E calls this prediction into question by stating that another factor
is causing consumers to hold onto cell phones for longer.

The filtering of hard water greatly reduces the number of gallstones and reduces
the severity of the gallstones that do occur. Filtering is profitable for the nation,
because it insures good health of the nations citizens. A legislator proposes that
communities should therefore be required to adopt filtering systems and to reach
the target of filtering 50 percent of all tap water within 5 years.

Which of the following, if true, most seriously calls into question the advisability of
implementing the proposal?

(A) Existing filtering systems programs have been private and voluntary, with
citizen participation ranging from 25 percent in some communities to 70
percent in others.

(B) Existing filtering programs have been restricted to the tap water that is ingested by
humans.
(C) Existing filtering programs have had recurrent difficulties selling their filtering
systems privately usually because the quantities filtered are too small to be
affordable for most families.
(D) Some of the materials filtered out of the water are materials that can be used for
other purposes.
(E) There are four different methods of filtering water and it is not clear which is the
most efficient.
The best answer is B. Filtering all tap water will be an ineffective method of
preventing disease is a large percentage of the water filtered is not meant for
human consumption.

A recent survey in country X found that more VCRs than videotapes were purchased
by its citizens last year. The best interpretation of this finding is that the practice of
illegally duplicating commercial videotapes is widespread.
Which of the following, if true, would cast doubt on the conclusion advanced
above:
(A) Very few VCR owners in country X film their own videotapes.

(B) There are few free noncommercial videotapes available in country X.


(C) The government of country X regularly copies and distributes of material it
considers to be classic content videotape free of charge, and to which the country
owns copying and distribution rights.
(D) The typical country X VCR does not purchase commercial videotapes in other
countries.
(E) Many citizens of country X legally make duplicates of videotapes, but such
duplicates were counted as purchased copies in the survey.
The best answer is C. If C is true, it could account for the discrepancy between the
number of VCRs and the number of videotapes.
A local board of education has decided to allow its Spanish teachers more
freedom to customize the curriculum for their classes. The teachers can edit out
those sections of the curriculum they are not interested in and add material of
their own choosing.

The widespread use of the option mentioned above is LEAST likely to contribute to
fulfilling which of the following educational objectives?
(A) Coverage of material relevant to a particular a student specific needs
(B) Coverage of basic grammatical structures and the exceptions to these grammatical
structures.
(C) Ensuring that students nationwide engaged in a specific course of study are
uniformly exposed to a basic set of readings
(D) Making the textbooks used in classes more satisfactory from the individual
teachers point of view
(E) Keeping students interest in a course by offering lively, well-written reading
The best answer is C. By allowing the teachers leeway in the curriculum, students
nationwide are not likely to be exposed to the same material.
Companies in the country of Kabbar sell transistor radios in the country of Maldena at
a price that is below what it costs Maldenian companies to produce them. To help
those Maldenian companies, the Maldenian legislature plans to set a minimum selling
price for transistor radios imported into Maldena from Kabbar that is twelve percent
greater than the average production costs for companies in Maldena.
Which of the following, if true, most seriously threatens the success of the plan?
(A) The annual rate of inflation in Kabbar is expected to exceed twelve percent within
the next year.
(B) Maldena is not the only country where Kabbar manufacturers currently sell
transistor radios.
(C) Some Maldenian companies that sell transistor radios have announced that they
plan to decrease their price for transistor radios.
(D) The government of Kabbar will also set a minimum price for selling transistor
radios in their country.
(E) Emerging companies in countries other than Kabbar will be able to sell transistor
radios in Maldena at a price below what it costs Maldenian companies to
manufacture them.
The best answer is E. The objective of the plan is to protect local Maldenian
companies. The government plans to do so by ensuring that transistor radios
from Kabbar do not undersell the local product. Choice E states that other
countries would still be able to sell more cheaply than local manufacturers and
this seriously threatens the success of the plan.

Bus Company President: A city councilman has stated his objection to the claim
that the city buses are dangerous. This is clearly untrue. None of the new buses
in use has ever crashed or even had a serious malfunction.

City councilman: The problem with the new buses is not that they malfunction, but
that they are so flamboyantly painted that other drivers on the road are distracted and
this may cause accidents.
The councilman responds to the bus company president by doing which of the
following?
(A) Characterizing the bus company presidents assertion as stemming from
subjective interest rather than from objective evaluation of the facts
(B) Drawing attention to the fact that the bus company presidents interpretation of
the word dangerous is too narrow
(C) Invoking evidence that the bus company president has explicitly dismissed as
irrelevant the point at issue
(D) Citing statistical evidence that refutes the bus company presidents claim
(E) Casting doubt on the extent of the bus company presidents knowledge of the
number of recent traffic accidents
The best answer is B. The bus company president counters the claim that the
companys buses are dangerous by stating that their buses have not been
involved in an accident or malfunctioned. The councilman in turn states facts that
show how the buses can indeed be dangerous, but not in the way the president
means.

Certain upgrades need to be made on each of the computers owned by Bosco


Insurance. The work of upgrading their 20 computers is mentally taxing.
Employees are using the computers during the day, making it difficult for the
computer technician to coordinate the block of time needed to work on each
computer. Therefore, the most efficient time to upgrade the computers is
between ten oclock p.m. and eight oclock a.m. when the computers are free.

Which of the following, if true, argues most strongly against the view that upgrades
could be done more efficiently by scheduling them at night?
(A) Energy costs in businesses are greatly lower at night than they are during the day.
(B) More blackouts occur between midnight and eight oclock in the morning than
at any other time.

(C) Over the course of a year, the companys computers will require periodical checks
and upkeep

(D) Computer technicians are generally paid the same when they work during the
night than when they work during the day
(E) Mental alertness is lower in the late night than during the day, especially in people
not accustomed to working at night.
The best answer is E. The conclusion is that night is the most efficient time for
the computer technician to work. However, if the technicians mental alertness
will be lower, this cast serious doubt that this task could be done most efficiently
at night.

Extracorpreal shockwave lithotripsy a noninvasive procedure can be used to


remove painful kidney stones. In contrast to surgery the invasive procedure
customarily used extracorpreal shockwave lithotripsy poses little risk to patients.
Thus, to guarantee patient safety in the attempt treat kidney stones, extracorpreal
shockwave lithotripsy should replace kidney stone surgery in all cases.
Which of the following, if true, would most support the recommendation above?
(A) Kidney surgery can be used to treat other ailments of the kidneys other than
kidney stones.

(B) Extracorpreal shockwave lithotripsy was designed primarily in order to diagnose


kidney stones.
(C) Kidney surgery can reveal more information about the cause of a stone than an
extracorpreal shockwave lithotripsy can.
(D) An extracorpreal shockwave lithotripsy is just as likely as kidney surgery to
successfully remove a kidney stone.
(E) Some patients for whom surgery presents no risk are unwilling to undergo
extracorpreal shockwave lithotripsy.
The best answer is D. If both procedures have the same rate of success, the
conclusion that the safer procedure, extracorpreal shockwave lithotripsy, should
be used in all cases, is supported.

A sign in an eye clinic advises people who have received eye drops that blur vision to
refrain from driving until they can do so safely. In a hospital study, however, subjects
questioned immediately after they received such drops underestimated the time
necessary to regain their driving ability. This result indicates that many people with
eye drops will be unlikely to follow the advice.
Which of the following, if true, most strongly supports the argument above?

(A) Many people, if they know they will receive eye drops, make arrangements
beforehand for a friend or relative to drive them home.
(B) The subjects in the hospital study generally rated their abilities more
conservatively than they would if they were outside the framework of a scientific
experiment.
(C) Some people ask their doctors not to give them eye drops that blur vision if they
will have to drive to get home afterward.
(D) The subjects in the hospital study were also questioned about the time necessary
to regain abilities that do not play an important role in driving safely.
(E) Awareness of the sign is higher among the general population than it was among
the subjects in the hospital study.
The best answer is B. The sign advises people to drive only when they are able to
do so. According to choice B, patients are unable to determine correctly when
they are fit to drive. This supports the conclusion that many people will not
follow the advice on the sign.

Both Superfit and Fun-Fit have printed a toll-free number on their exercise
products to provide round-the-clock telephone assistance to any customer who
encounters difficulties. Since customers only call the numbers when they find the
exercise equipment difficult to use, and the Superfit line receives three times as
many calls as the Express line, Superfits product must be more difficult to use
than Fun-fits.

Which of the following, if true, most strengthens the argument above?


(A) Calls to the Fun-Fit number are almost twice as long, on average, than calls to the
Superfit line.
(B) Fun-Fit has sold four times the number of its exercise product that Superfit has.
(C) Fun-Fit receives twice as many letters of complaint about its product as Superfit
receives about its product.
(D) The number of calls received by each of the two lines has been gradually
increasing.
(E) The Superfit number is more widely publicized that the Fun-Fit number.

The best answer is B. The passage argues that since Superfit receives more
telephone calls, the product must be more difficult to use. If Fun-Fit has sold four
times the number products as Superfit has sold, this claim is strengthened.

Reporter: Argent Antiques has misled its clients by promoting some Chinese porcelain
figurines as rare when in fact those pieces are relatively commonplace.
Argent dealer: That is absurd! Argent Antiques is the largest antique dealer in the
country. We authenticate the figurines we sell through a nationally recognized firm.
The Argent dealer s reply is most vulnerable to the criticism that it
(A) exaggerates the reporters claims in order to make them appear absurd
(B) accuses the reporter of bias but presents no evidence to support that accusation
(C) fails to establish that other antique dealers do not also authenticate the figurines
those dealers sell
(D) lists strengths of Argent Antiques while failing to address the reporters charge
(E) provides no definition for the inherently vague phrase rare
The best answer is D. Argent antiques has been accused of promoting some pieces as
rare when they are, in fact, common. The dealers reply does not directly address this
accusation, rather it only lists the companys strengths.

Potrebbero piacerti anche